You are on page 1of 44

TORTS OUTLINE

GENERAL NOTES
Negligence Cases:
Jury does 2 things:
o Makes a finding on fact
o Makes law between 2 and 8 A jury with 20/20 hindsight gets to set the standard of care and the average
person cannot know beforehand.
Look at scale of cases from 1-10. If the case is between 2-8 it goes to the jury. If it is appeals, and the appeals
court affirms the decision, they are not affirming the policy, they are affirming the fact that this is a 2-8 case and it
is a jury decision. Means it is okay for the jury to set the standard of care, but it does not mean that a different jury
could not have had a different answer. Sets a precedent that it is not directed verdict material. No consistency in
negligence cases.
Directed Verdict If the case is below a 2 or above an 8, the case will not get to a jury. A judge will decide the
case on the grounds that reasonable persons cannot differ.
If the court grants a directed verdict and it stands on appeal the court has made a decision on the facts that this
kind of standard of care is clear.
Directed Verdict some are very close. 19 judges vs. 17 judges. How can we say that reasonable persons cannot
differ? Actually saying what the acceptable standard of care is. Making policy.
CHAPTER 1:
INTENTIONAL TORTS: INTERFERENCE WITH PERSONS AND PROPERTY
Intent:
2 Approaches:
1. If defendant acts with intent, it does not matter if the probability of harm or offense is low liability is imposed if
what was intended occurs.
2. If the defendant does NOT intend contact, there must be near certainty that it will occur.
The state of mind and what you know matters.
1. purpose to cause contact or apprehension (and does not matter if probability of injury is low if you have intent);
OR
2. knowledge to substantial certainty to hurt and offend
3. knowledge to substantial certainty that contact or apprehension will take place (Dailey definition)
Restatement- for liability for battery
an act which, directly or indirectly, is the legal cause of a harmful conduct with anothers person makes the actor liable to
the other, if
(a) the act is done with the intention of bringing about a harmful or offensive conduct or an apprehension
thereof to the other or a third person, and
i. character of actors intention- act must be done for purpose of causing the contact or
apprehension or with knowledge on part of actor that is substantially certain to be produced
ii. negligence or recklessness is not enough- must be intentional
(b) the contact is not consented to by the other or the others consent thereto is procured by fraud or duress,
and
(c) the contact is not otherwise privileged
1.

Garratt (P) v. Dailey Five year old boy pulled the chair out from under Garratt. Liable for battery.
a. Court:
i. Absence of intent to cause injury does not absolve liability if it was known to defendant that
plaintiff would try to sit where chair had been and his action would cause plaintiff to fall to the
ground.
ii. Established that he did not have intent to hurt, but did not look at the second prong.
iii. Case is remanded to see if Brian knew to substantial certainty that Garratt was going to try to sit
down.

b.
c.
d.

In other words, know that the offensive contact would take place. If he had intent to make contact (and
not intent to harm) then he is liable.
Infants are responsible for intentional torts (not negligence)
Contact between P and D can be indirect.

mere intent to contact rule- some courts adopt this


o P must prove the D intended the contact to be harmful or offensive to the person
o Must show intention to contact and intention to harm
o Twerski likes this- need to intend harm or be substantially certain that harm will occur

2.

Transferred Intent
a. Defendant intends to commit a tort against one person, but commits the same tort against a different
person.
b. Can only be transfer of intent if the original intent was intended to be tortiuous.
c. Applicable to assault, battery, false imprisonment, trespass to land and trespass to chattels. Not selfdefense.

3.

Ranson v. Kitner (1808) HAD INTENT TO CAUSE A CONTACT, GOOD FAITH DOES NOT MATTER.
Hunting for a wolf and shoots a dog that resembled a wolf.
a. He is liable because of his intent to hit an object regardless of what the object is. No need to intend to do
something harmful. Acting in good faith and still held liable.
b. Knowledge to substantial certainty that you were going to make CONTACT.
c. Guilty for his intent to contact.

4.

Talmage v. Smith CAN USE REASONABLE FORCE TO PROTECT PROPERTY AND NO TRANSFERRED
INTENT.
(1894) Smith sees 1 kid on top of his shed, throws a stick that hits Talmage, Smith clearly did not see Talmage
when he threw the stick- threw stick to scare, not to hit anybody
a. Smith did not intend to hit anybody- cant use transferred intent doctrine
b. Cannot use doctrine of transferred intent because original conduct was not tortious.
c. Using reasonable action to get trespassers off his property.
d. No intent, and yet a battery, and Talmage cannot recover.
e. Boy was an unforeseeable plaintiff Mr. Palsgraff.

5.

Intent in other contexts:


a. Insurance Policies Exclude intentional torts to prevent moral hazard.
b. Workers Compensation If you are injured on the job you will recover all economic losses (medical,
loss of earning, out of pocket expenses) without having to prove intent or negligence or that you were not
at fault CATCH you cannot recover pain and suffering or intentional tort.
i. Ex Dangerous milling machine slow with safety guard, employer takes out safety guard WITH
KNOWLEDGE TO SUBSTANTIAL CERTAINTY that injury would result. D can be liable.
(Holtz v. Schutt Pattern Works)

Battery
1. Un-permitted contact upon the person of another which is harmful or offensive.
2. Defendant INTENDS to bring about contact that is harmful or offensive to the reasonable person. (Do not need
intent to harm).
3. Must make contact this can extend to objects associated with the victims body
4. Even if motive is good (help someone who fell) if you intend to cause a contact that you know is unwanted
battery
5. Contact that is offensive to the reasonable person.
6. Causation
a. Direct contact OR
b. Indirect contact set in motion a force that brings about a harmful or offensive contact to the plaintiffs
person. Contact with clothing or other object.

c. Actual damage not required.


Restatement 13 Battery: Harmful Contact
An actor is subject to liability to another for battery if
(a) he acts intending to cause a harmful or offensive contact with the person of the other or a third person, or an
imminent apprehension of such a contact, and
(b) a harmful contact with the person of the other directly or indirectly results
Restatement 18 Battery: Offensive Contact
(1) An actor is subject to liability to another for battery if
(a) he acts intending to cause a harmful or offensive contact with the person of the other or a third person, or an
imminent apprehension of such a contact, and
(b) an offensive contact with the person of the other directly or indirectly results
(2) an act which is not done with the intention stated in Subsection (1,a) does not make the actor liable to the other for a
mere offensive contact with the others person although the act involves an unreasonable risk of inflicting it and, therefore
would be negligent or reckless if the risk threatened bodily harm.
1.

Brzoska v. Olsen CONTACT MUST OFFEND A REASONABLE SENSE OF PERSONAL DIGNITY, AND
NOT HYPER-SENSITIVE PERSON (no channel for HIV so they were being hyper-sensitive).
38 patients of Dr. Owens sue for battery because Dentist had AIDS.
a. The intent necessary for battery is the intent to make contact with the person, not the intent to cause
harmthe contact need not be harmful, it is sufficient if the contact offends the persons integrity- must
offend a reasonable sense of personal dignity
b. Dr. Olsen cannot sustain a battery charge because there was no channel for the HIV infection (less than
1%). The fear of contracting AIDS is per se unreasonable and the patients were not exposed to the
disease
c. Court said contact was only offensive if results in actual exposure to HIV
d. Must look at reasonableness of the fear of the contact and it was unreasonable for the patient to fear
infection when not exposed to the disease. In other words, contact is offensive only if it results in
actual exposure to the HIV virus.
e. Class Discussion: What about the 6 months the patients had to wait to learn if they were infected with the
disease??? Twerski would consider this a battery.- recovery for window of anxiety period
f. Court ruled in favor of defendant with ulterior motive, did not want to perpetuate the stigma of AIDS.

2.

Plaintiff with an eggshell skull a peculiar vulnerability to injury may recover all his damages
notwithstanding the lack of foreseeability of the ultimate injury.
a. With battery once you are in for a nickel you can go all the way!

3.

Can a plaintiff recover for a slight contact? Professor lightly touches a student on her fully clothed shoulder.
The student brought a suit against the professor because she said the touch to her shoulder flooded her with
memories of being raped and molested when 11 years old.

4.

Fisher v. Carrousel Motor Home A CONTACT DOES NOT HAVE TO TOUCH PERSONS BODY, CAN
ALSO TOUCH ITEMS INTIMATELY ASSOCIATED WITH PERSONS BODY.
Flynn snatched a plate from plaintiffs hands and said he could not be served because he was black.
a. Fisher is really suing for emotional distress from the racial slurs.
b. Example of a battery without assault.
c. Battery because the plate was an object intimately connected with Fishers body- actual physical contact
is not necessary to constitute a battery, as long as there is contact with clothing or an object closely
identified with the body
d. Formal requirements of battery of flexible. Tort law is also affected by societal pressures and changes.
e. Use battery because intentional infliction of emotional distress was not born yet. Battery is an easier tort
to make out.
f. if employee is acting w/in course of his employment as a manager, then the corporation is liableemployer liable for employee when
principle authorized doing and manner of the act or
agent unfit and principle reckless in employing him or

agent acting in managerial capacity and was acting in the scope of employment or
employer/manager of employer ratified or approved act

Assault
Acts intending to cause a harmful or offensive contact and the other is put in such imminent apprehension of that
conduct
Assault involves a D who touches the mind of the P
Person has to be aware of intended harmful or offensive conduct
Need to prove that had intent to put in apprehension. Even if just felt in apprehension this is not enough. If
you cannot prove that had intent than all you have is negligent infliction of emotional distress.
Defendant touches the mind of the plaintiff.
An assault is just an incomplete battery Although true in many cases, in some cases a person swings and
victim does not see so not in apprehension of bodily contact no assault.
Generally, words alone are not enough to constitute an assault because they cannot create a reasonable
apprehension of immediate harmful or offensive contact. Different result if words are accompanied by an overt
act, like a clenched fist.
Restatement 1. An actor is subject to liability to another for assault if
a. He acts intending to cause a harmful or offensive contact with the person of the other or a 3rd person, or
an imminent apprehension of such a contact, and
1. offensive contact does not require battery.
2. imminent does not mean instantaneous just close enough that can happen in short period
of time. Threats relating to far away future are not enough.
b. The other is thereby put in such imminent apprehension.
1.

Western Union Telegraph Co. v. Hill ASSAULT TURNS ON WHETHER THE DEFENDANTS ACT
WOULD PLACE A REASONABLE PERSON IN APPREHENSION OF AN UNWANTED CONTACT, NOT
ON WHETHER THE AGGRESSOR IS IN FACT ABLE TO MAKE THE THREATENED CONTACT.
Employee made a sexual advance of Hills wife.
a. P cant recover b/c intent was not proven affirmatively
b. Employee attempted to put his hand on Hills wife by reaching out over counter, but the counter was very
high and he could not have touched her if he tried.
c. This might not be a battery if completed.
d. Could she be put in immediate apprehension of contact? Jury issue and the jury said yes so it was an
assault. Size of the counter really does not matter.
e. It does not matter how scared she was, the jury decides the intent of the defendant. look at rolling of
sleeves, or steps forward to determine intent.
f. Employee only has to intend to put her in apprehension of immediate contact.

False Imprisonment
Defendant has to intend to imprison and the plaintiff must feel imprisoned. (Restatement)
Plaintiff does not actually need to be confined; a locked space is not necessary, plaintiff just needs to believe.
The defendant must know that he or she will get reaction out of plaintiff.
1. Elements of false imprisonment:
a. A willful detention
b. Without consent
c. And without authority of law.
1.

Grant (P) v. Stop-N-Go Market Shopkeeper detains plaintiff because he thinks he stole a pack of cigarettes but
he is wrong.
a. Shopkeeper intended to scare Grant and Grant rightfully felt he could not leave.
b. Shopkeeper accused plaintiff of stealing, grabbed his arm, told him to shut up because he was going to
call the cops. Grant felt that if he left, the cops would come after him.

c.
d.

2.

3.

Shopkeeper believed he was going after a thief but he was wrong. RANSON which says that you are
guilty.
Notes: if the defendant subjectively believes that the plaintiff is willing to stay, then the defendant may
not intend to deprive the plaintiff of her freedom of movement, regardless of whether the plaintiff is or is
not intimidated by the circumstances

Shopkeepers Privilege Statutes or common law give shop keepers the privilege to detain for investigation
regardless of whether the detainee has actually stolen something or not. For the privilege to apply, must meet the
following conditions:
a. Must be a reasonable belief as to the fact of theft.
b. The detention must be conducted in a reasonable manner (no deadly force)
c. The detention must be for a reasonable period of time and only for the purpose of the investigation.
i. What is a reasonable amount of time?
1. Jaque v. Sears Roebuck Reasonable time includes the amount of time for the police to
arrive.
2. NY Statute Reasonable amount of time is the amount of time needed to make a
statement and to examine employees and record of the establishment.
d. Takes away the chance of Ranson. Allows you to make a mistake and get off the hook.
In cases of employers with employees- the threat of being fired, or the desire of an employee to clear his name do
not create grounds for false imprisonment
a. Fact sensitive
b. Was employee made to believe she couldnt leave?

Intentional Infliction of Emotional Distress


First time we define a tort not by the conduct, but by the severity of the result.
Courts have always allowed for recovery for emotional distress from battery, assault, and false imprisonmentwhen emotional distress is freestanding is when it gets difficult
a. First, courts recognized cause of action for IIED when actor should have foreseen that the emotional
distress would lead to physical harm
b. CA then gave IIED its own cause of action
Hotels, innkeepers, restaurants are held to a higher standard of care have a special duty.
Known sensitivity If defendant knows that plaintiff is more susceptible to emotional distress liability will follow.
Also liable for RECKLESS conduct.
1. Four elements: (Womack Test)
a. The conduct must be intentional or reckless (jury decides)
Must act with intent of causing mental distress
b. The conduct must be extreme our outrageous (jury decides)
Insults generally dont qualify
c. There must be a causal connection between the wrongful conduct and the emotional distress.
d. The emotional distress must be severe.
2. Restatement: Outrageous Conduct Causing Severe Emotional Distress
a. One who by extreme and outrageous conduct intentionally (purposely) or recklessly causes severe
emotional distress to another is subject to liability for such distress, and if bodily harm to the other results
from it, for such bodily harm.
3.

Harris (P) v. Jones Employer mocked the stuttering of employee over a period of five months to the point that it
drove him to drink.
a. No evidence of causal connection between Jones conduct and Harris distress does not even address the
other elements of the tort. No proof that result was serious enough.
b. There could be severe and outrageous conduct without severe emotional distress. Why isnt that enough?
It would allow people to fake it.

Trespass to Chattels and Conversion

Even if you make a mistake and act reasonably, if you step on another persons property than there is a tort.
Authority is RANSON.
a. No intent to trespass needed, only intent to be on land
Conversion Plaintiff has the option of taking back chattel and paying defendant for damage done OR defendant
keeps chattel and pays plaintiff the fair market value.

1.

Trespass to land - Just need intent to go on property, do not need intent to trespass. Reasonableness does not
matter, no gray areas.

2.

Trespass to chattels: An intentional interference to personal property of another although only minor.
without some damage to the chattel there is no cause of action for trespass to chattels

3.

Restatement definition of Conversion:


a. Conversion is an intentional exercise of dominion or control over a chattel, which so seriously interferes
with the right of another to control it that the actor may justly be required to pay the other the full value of
the chattel.
b. In determining the seriousness of the interference and the justice of requiring the actor to pay the full
value, the following factors are important:
i. The extent and duration of the actors exercise of dominion or control
ii. The actors intent to assert a right in fact inconsistent with the others right of control
iii. The actors good faith
iv. The extent and duration of the resulting interference with the others right of control.
v. The harm done to the chattel
vi. The inconvenience and expense caused to the other.

4.

Rogers v. Kent Board of County Road Commissioners Even though acted innocently when had intent to put
in stake, and negligent when didnt take it out, go back to original intent to make tort.
Defendant left post in ground and failed to remove it. Decedent mowed over post (it was covered by grass) and as
a result of the impact was thrown to the wheels of the mowing machine.
a. Defendant acted with intent to put stake in ground, but that is not tortuous.
b. No need for intent to Trespass to Chattels.
c. a trespass may be committed by the continued presence on the land of a structure or other thing which the
actor put there legally with the consent of the person in possession of the land- but fails to remove it when
consent has been terminated
d. TWERSKI sees a breach of contract claim, but wonders why it is a trespass- the intent to put it in was
fine
e. Another situation like RANSON- where innocent intent (or negligence) will make you liable for an
intentional tort

5.

Compuserve v. Cyber Promotions, Inc. Plaintiff tries to stop defendant from sending SPAM.
a. SPAM e-mail is trespass to chattels because it slowed down the server (damage).
b. Many hypotheticals. At some point annoying phone calls could be trespass to chattels (assuming you
suffer some kind of damage).
c. Glidden v. Szybiak Action for trespass to chattels cannot be maintained without some form of damage.

6.

Intel v. Hamidi Plaintiff sues D for sending unwanted emails through their system
a. Did emails damage or interfere with their system?- was there actual injury?
b. Decisions finding electronic contact to be a trespass to computer systems have generally involved some
actual or threatened interference with the computers functioning
c. Though not amounting to conversion, the Ds interference must, to be actionable, have caused some
injury to the chattel or to the Ps rights in it
d. Injury can only be established by the completed torts consequences, not by the costs of the steps taken to
avoid the injury and prevent the tort

7.

Umbrella Intentional Tort Draft states, An actor who intentionally causes physical harm is subject to liability
for that harm.
a. Essentially says If you intentionally cause physical harm that you are liable harm.
b. Is this really a good idea?
i. Would add liability for some infliction of emotional distress torts that are not severe enough.

CHAPTER 2:
PRIVLEDGES/DEFENSES
Consent- consent operates to eliminate the Ps prima facie case- the offensiveness of the contact
1. Restatement: Meaning of Consent
1. Consent is willingness in fact for conduct to occur. It may be manifested by action or inaction and need
not be communicated to the actor.
2. If words or conduct are reasonably understood by another to be intended as consent, they constitute
apparent consent and are as effective as consent in fact.
2. What did the person mean and what did the other person understand?
1. raising arm
2. getting on football field
3. consenting to operating table.
3. to one who consents, no harm is done
4. Privilege Able to act in a certain way when does not have a legal duty to refrain from so acting.
5. Sometimes legislature has banned consent minor cannot consent to sex so defendant is also banned from
consent.
1.

OBrien v. Cunard Steamship Co. IS PRIVILEGED WHEN S CONDUCT REASONABLY MANIFESTS


CONSENT.
OBrien was vaccinated when she had already received the vaccination and became sick again.
a. Who has the duty to ask or give consent? How much consent is enough? Largely a jury issue.
b. Girl raised her arm
i. She might have been showing that she already received the shot.
ii. Doctor might have thought this was a sign of consent.
c. If she communicated consent then she waived her right.
d. Judgment for defendant because court did not want to explore question of how much consideration is
enough to merit consent.
e. This was considered a reasonable mistake

2.

Hackbart v. Cincinnati Bengals, Inc. p. 61 Intentional striking of a player while the player is down the minute
after the whistle blows.
a. How much does an individual consent to in institutional settings? When you step onto a football field,
you consent to a whole lot of battery. BUT, did you consent to this instance of battery?
b. Appellate court remanded and lower court was told to figure out the extent of consent. Lower can do
nothing but find for the plaintiff.
c. TWERSKI: the parties are in perfectly good shape to contract this kind of thing out

3.

Christmas v. Davis Supp. 2- Doctor was in the midst of periodontal surgery and decided to do less intrusive
surgery than originally agreed upon. P sued for malpractice and battery.
a. The battery theory should be reserved for those circumstances when a doctor performs an operation to
which the patient has not consented
b. When the patient consents to certain treatment and the doctor performs that treatment but an undisclosed
inherent complication with a low probability occurs, no intentional deviation from the consent given
appears
c. TWERSKI: the guy wanted the more serious surgery, giving him the less serious one was something he
didnt want- should make out a battery
Mohr v. Williams (p. 70)- even Dr who performs well, can be liable for battery when consent was not given

4.

5.
6.

7.

Perry v. Shaw (p. 71)- P says no to surgery, then consents while medicated- no consent- this is battery
DeMay v. Roberts Physician brought non-medical person to birth house.
a. Consent to come in was clear, communication in the other direction was the problem.
b. How much does defendant have to tell plaintiff before plaintiff can give effective consent?
c. This became the tort of battery when he touched her, but he was saving her from falling off the bed.
d. Does doctor have a duty to tell plaintiff that he is not board certified?
e. Consent came by way of deceit- appeared as if he was medical personnel
f. Twerski: only touched her in an emergency, P could have asked if he was really there for medical
purposes- could be contributory fault on part of P for not inquiring who the man was
Consent Invalidated by Fraud- Restatement
a. Misrepresentation must be about something that affects the intrinsic nature and quality of the invasion or
harm. Misrepresentations about collateral facts will not invalidate consent.

8.

Informed Consent If physician obtains a patients consent but has breached his duty to inform, the patient has a
cause of action sounding in negligence for failure to inform the patient of his options, regardless of the due care
exercised at treatment, assuming there is an injury.
a. How much information is a doctor supposed to give a patient about a medical procedure?
b. Materiality 2 possible tests:
i. Reasonable patient test JURY DECIDES
1. A risk is material if a reasonable patient would want to know the information before the
procedure.
2. Does not need a medical expert to make out a case. Just get doctor on the stand he will
have to say there is a risk when performing a certain procedure. Doctor will be guilty
of negligence.
3. P must show if he had been aware of risks, he wouldnt have consented.
4. The adverse consequences not made known did in fact occur and P was injured as a
result of undergoing the treatment.
ii. Reasonable doctor test Medical experts determine the reasonable standard.
1. Custom of what physicians disclose is the test.
2. Need a medical expert- if they testify this doesnt meet medical custom, the doctor is
negligent
iii. Canteburry v. Spence Uses the reasonable patient test.
iv. NY Pubic Health Law- adopted a tough cause of action for lack of informed consent: applies the
professional community standard and specifically provides that "[l]ack of informed consent
means the failure... to disclose to the patient such alternatives... and the reasonably foreseeable
risks and benefits involved as a reasonable medical... practitioner under similar circumstances."
c. Decision causation the patient would have chosen a different option.
d. Injury Causation apportioning damages is also difficult because patient would have had other risks
with alternate surgery so cannot make patient whole. Damages are usually be what happened as a result
of non-consented intervention. Hernia case.
e. Courts will usually say lack of informed consent is negligence, not battery
f. Twerski argues that courts should recognize a duty on the part of hospitals to reveal to patients statistical
info regarding the relative risks associated with individual medical providers and if this isnt provided
then patients have cause of action for lack of informed consent

9.

Scott v. Bradford (D) No decision causation so no recovery.


Bradford performed a hysterectomy and Scott experienced problems with incontinence after surgery. Scott signed
a routine consent but Bradford did not properly advise her of the risks and if she was properly informed she would
not have had surgery.
a. Rejected the reasonable doctor test.
b. Accepted the reasonable patient test. The scope of a physicians communications must be measured by
his patients need to know enough to enable him to make an intelligent choice. Whether a risk is
immaterial is a question of fact for the jury.
c. A causal connection between the patients injury and the doctors breach of a duty to disclose exists only
when the disclosure of material risks would have resulted in a decision against procedure.

d. For decision causation - Canterbury v. Spence uses a reasonable patient test. This court rejects
that and looks at the actual patient. Says looking at the reasonable patient undermines selfdetermination.
e. The final element is that of an injury. The risk must have actually materialized, AND P must have been
injured as a result of submitting to the treatment.
f. Oklahoma says informed consent is a negligence case and no longer a battery.
g. There is no need to disclose obvious risks or risks already known to patient.
h. 3 requirements: 1. failure to inform of a material risk. 2. decision causation 3. the harm that results
is the harm that was not warned of.
i. TWERSKI: says makes no sense b/c a reasonable patient would always listen to a reasonable doctor
10. Canesi v. Wilson Doctor did not tell pregnant woman about risks of Provera when a warning in the PDR said it
should not be taken in the first 4 months of pregnancy because it could cause harm to fetus and limb reduction.
Turns out it does not cause limb reduction.
a. Elements of informed consent:
i. Failure to inform Doctor conceded negligence.
ii. Causation Court said you have not made out the elements of inform-consent because you
cannot prove causation.
b. Wrongful Birth Based on fact that doctor took away mothers choice to abort. Do not need to prove
causation.
c. Decision causation was made out but injury causation was not.
The case stands for the fact that if you proving informed-consent then you NEED CAUSATION. But then it backs
off and says mother was deprived of a choice, and that is an independent cause of action.
Self-Defense
Self-defense must be in response to an imminent threat.
Defenses fit in categories self-defense, defense of others, necessity (partial defense).
1. Restatement:
a. Self-Defense by Force Not Threatening Death or Serious Bodily Harm:
i. Actor is privileged to use reasonable force, not intended or likely to cause death or serious
bodily harm, to defend himself against unprivileged harm or offensive contact or other bodily
harm which he reasonably believes that another is about to inflict intentionally upon him
ii. Self-defense is privileged under the conditions stated in Subsection (1), although the actor
correctly or reasonably believes that he can avoid the necessity of so defending himself,
(a) by retreating or otherwise giving up a right or privilege, or
(b) by complying with a command with which the actor is under no duty to
comply or which the other is not privileged to enforce by the means threatened
b. Self-Defense by Force Threatening Death or Serous Bodily Harm:
i. Actor is privileged to use force intended or likely to cause death or serious bodily injury when he
reasonably believes that:
(a) the other is about the inflict upon him an intentional contact or other bodily
harm, and that
(b) he is thereby put in peril of death or serious bodily harm or ravishment, which
can safely be prevented only by the immediate use of such force.
ii. the privilege stated in Subsection (1) exists although the actor correctly or unreasonably believes
that he can safely avoid the necessity of so defending himself by
(a) retreating if he is attacked within his dwelling place, which is not also the
dwelling place of the other, or
(b) permitting the other to intrude upon or dispossess him of his dwelling place, or
(c) abandoning an attempt to effect a lawful arrest
iii. the privilege stated in Subsection (1) doesnt exist if the actor correctly or reasonably believes
that he can with complete safety avoid the necessity of so defending himself by
(a) retreating if attacked in any place other than his dwelling place, or in a place
which is also the dwelling of the other, or

(b) relinquishing the exercise of any right or privilege other than his privilege to
prevent intrusion upon or dispossession of his dwelling place or to effect a
lawful arrest
D can make a reasonable mistake in self-defense

2.

Courvoisier v. Raymond ACTOR MAY USE DEADLY FORCE IF REASONABLY BELIEVE NECESSARY
Defendant was protecting his house from robbers when a police officer stepped forward and defendant shot the
police office. Instruction of trial judge erroneous.
a. Transferred intent does not apply to self-defense.
b. intended to stop attacker and NOT innocent bystander.
c. Duty to retreat if safe was in house so no duty.
d. Similar to Ranson but defendant had a special privilege so his actions are excusable.
e. Similar to Talmage using reasonable force and acting in response to an invasion.

3.

Katko v. Briney CANNOT USE DEADLY FORCE TO PROTECT PROPERTY FROM


TRESPASSERS/CRIMINALS.
Defendant set up a shot-gun trap and plaintiff entered with intent to steal cans and plaintiffs leg was blown away
and permanently deformed.
a. Twerski does not think a sign would have mattered because restatement blasts There is a higher value on
human safety than property.
b. What about guard dogs? They are different because you hear them, but also similar because they are
there to protect property when residents are not home.
c. It is okay to take reasonable measures to protect our property (Talmage) but you cannot use deadly force.
d. Mere trespass against property other than a dwelling is not a sufficient justification to authorize the use of
a deadly weapon by the owner in its defense.
e. Posner Courts adjudicating defense-of-property matters should rely upon case-by-case cost-benefit
analysis and look at location, type of warning, difficulty in protecting.

4.

Recovery of Property You have a right to use reasonable force to regain chattel tortiously taken by another so
long as you act in a timely fashion. If you are wrong, even if you acted reasonably, you are no longer privileged
and can be held liable for battery.
a. Limits on self help privileges:
i. Once immediacy is lost- so is the privilege
ii. Reasonable force only- deadly force never permitted
iii. If the possessor seeking to reclaim is mistaken- privilege not available

5.

Vincent v. Lake Erie Transportation Co. IF YOU DAMAGE SOMEBODY ELSES PROPERTY TO SAVE
YOUR OWN YOU HAVE NOT TRESPASSED BUT MUST PAY DAMAGES.
Defendant tied vessel to dock and then a storm came and vessel could not be moved. Damage to dock resulted.
a. He acted reasonably; if he had left boat in the ocean it could have caused more damage.
b. Private necessity is an incomplete defense.
c. Court does not question right of guy to moor ship to dock. You have the privilege to use private property
in an emergency to save your own person or other people, but you might have to pay for victim.
d. What if he didnt choose a dock? We dont know.
e. Ship-owner is liable for damages to dock, but not consequential damages (dock owner out of business for
2 weeks cannot recover lost earnings).
f. This is a policy decision do not want to encourage dock owners to prevent ships from mooring during
storms because there could be greater damages.
g. Under this case, if someone has a heart attack in your house, he has a right to be there but if he damages
something you have a right to charge him.

10

Legal Authority- privileges retained by public officials

When such officials act w/in the limits of their predefined roles, exercising the power and authority that those roles
afford them, such officials are not subject to liability for those actions

- police officers are example

- parents and teachers to a degree


6.

Sindle v. New York Transit Authority P CANT RECOVER WHEN ACT UNREASONABLY IN REGARD
TO HIS OWN SAFETY
a. Bus was on its way to police precinct as kid jumps out and gets run over by bus.
b. Court said driver has a privilege but what is the privilege? Does not fit into a neat category. Could be
self-defense, defense of others, shopkeepers
c. Bus driver is different from Ranson because he was facing a threat, he was in a reactive mode.
d. Dangerous to not categorize because you do not want to send the message that there is a defense to every
intentional tort.
e. Although confinement reasonably perceived to be unlawful may invite escape, the person falsely
imprisoned is not relieved of the duty of reasonable care for his own safety in extricating himself from the
unlawful detention.
f. Holds that there is a privilege of justification- has duty to take reasonable measures for protection of
passengers and property- jury shouldnt have excluded the evidence bearing on the justification issue- if
the D acted reasonably, then he gets off the hook (like Ranson).

CHAPTER 3:
NEGLIGENCE

Lots of steps for negligence:


o First, DUTY
In general, a duty of reasonable care to those around us
o if there was duty, was the duty breached?
Jury must set standard of care (was checking lights once a day sufficient?)
Then, must check the facts (did he check the lights once a day?)
Check the statute- if applicable- negligence per se- still need to ask if he was speeding
Jury sets the standard and then tells you if it was breached
Courts tend to talk in categorical terms. Have to be careful when speaking in overbroad terms. Example:
o Baltimore & O.R. v. Goodman Holmes made a rule that driver who cannot see approaching train has to
get out of car and look.
o Pokora v. Wabash Driver of truck did not get out and look, and if he did he would have been in greater
danger. Cardozo says the Holmes rule is not good law and making rules in negligence cases is dangerous
because each case requires a fresh look and might need to be sent to a jury.

Plaintiff must establish each of the FIVE elements:


1. Duty Members of society have a duty to act reasonably.
2. Breach of Duty Plaintiff must establish that the defendant failed to act reasonably.
3. Cause-in-Fact Causal connection between negligence and harm suffered.
4. Proximate cause Must be a foreseeable consequence.
5. Harm Plaintiff must suffer tangible harm.
Make allowances for (considered as circumstances
How would a reasonable person under the circumstances
act?
Children have to learn.
Disability

Do not make allowances for:


Only one reasonable person standard, keep the
subjective out of it.
Lacking good judgment
Acting in haste

11

Emergency
Custom
Statute
Expert Status

1.

Lubitz v. Wells Was Dad negligent when he left a golf club in the backyard where unattended kids were playing?
NO
a. Setting the reasonable standard of care - Common Rule Created You cannot be held negligent for leaving a
common object lying around UNLESS it is inherently dangerous.
b. There is NO DISCOVERY. Do not ask important questions that could change the situation.
c. Very high threshold no violins.
d. Create such a rule because you do not want to hold a parent negligent for kids action.

2.

United States v. Carroll Towing Co. Bargee left boat, tugboat rearranged moorings so it could clear an obstruction,
Anna C (the bargees barge) broke free, crashed into another tanker and sank.
a. 2 Questions:
i. Is the bargee negligent for leaving the boat?
ii. If so, than the bargees owner cannot recover 100%.
b. This is how to decide if someone is negligent:
B (Burden) < P (Probability of Accident) * L (Gravity of Loss or Harm)
i. Does not consider alternatives to avoid injury.
c. Question of how much should you invest to avoid an accident.
i. If Burden is high and Probability is low, you are probably not going to get to the jury.
d. In this case no place to get the data:
i. B - it would have been easy to avoid the harm have bargee there.
ii. P
iii. L hard to figure out, always a possibility that someone could have been hit but likelihood of dying
is low.

3.

Washington v. Louisiana Power & Light Co. Father electrocuted when he tried to move a radio wire and
accidentally allowed it to touch an un-insulated electric wire in his backyard.
a. Appellate court reversed a jury verdict in favor of plaintiff. Court says this risk is too small and this burden
(to put electrical wires underground) is too big. Directed verdict for the defendant- not negligent in failing to
get rid of the risk
b. Learned Hand Formula economic efficiency approach
i. B (Burden) expensive to bury miles of wire underground. Electric company deals with these little
risks everywhere and it would be a huge burden to take care of all of them.
ii. P (Probability of Accident) Low probability that someone would detach an antenna and carry it
near a power line.
iii. L Loss Death

4.

Reasonable Person An OBJECTIVE and a not a subjective test.


a. Want to take into account varying circumstances (age, skills, emergency reaction time) but if we subjectify the
negligence too much we destroy the standard for judging human conduct.
b. Vaughn v. Menlove Defendant knew that hay rick was risky and could spontaneously combust and tried to
build a chimney in the middle to reduce risk of fire, but fire happened. Defendant argued his good intentions
should be judged, but court said no.
i. He exercises best judgment and gets bad result. No sympathy. Bad judgment results in poor valuesis an objective not a subjective standard- go with standard a man of ordinary prudence would observe
ii. Establishes 2 questions:
1. What is the P for defendant? Whatever he knows the probability of harm to be.
2. SHOULD YOU HAVE BETTER DATA? Should you have acted differently because
you didnt know? How much should you spend to get more info?

Intelligence
Character
Mentally Ill

12

5.

Delair v. McADOO
Defendant was driving with vault tires (fabric) anybody could see they were worn down. Jury finds defendant
negligent, and appeals court affirms. What kind of precedent does this set?
a. Not a good case for BPL because L is unpredictable highly speculative business.
b. Negligence as a matter of law. Not saying that every person must know the condition of his tires, just saying
that this was a jury question (between 2-8) and we affirm what the jury said.
c. If you change facts (borrowed car) than you might have a different answer.

6.

Variations of the Reasonable Person


a. Do Reasonable Persons Forget? Ted Kennedy veered the wrong way in a fork in the road (a road that he had
driven on thousands of times) and ended up in bay. This is a jury issue.
b. Expert Knowledge If you have expert knowledge should you be held to a higher standard? Answer is
generally YES. Ability to evaluate and respond to risk is much different. However, question pops up when
you are not at work, and a reasonable person would have been found not negligent.

7.

Cordas v. Peerless Transportation Co. EMERGENCY DOCTRINE WHEN JUDGING THE


REASONABLENESS OF THE CONDUCT CONSIDER EMERGENCY. FACT THAT HAD TO ACT QUICKLY
IS RELEVANT.
Cabbie at gunpoint jumps out of cab while it is still running and it hits a mother and her 2 children.
a. Was Cabbie negligent when he jumped out? Court said NO.
b. Problems with B<PL You have terrible data to assess the P of the B<PL and you do not have time to get
better data. Therefore, there might have been a better decision but it is hard to second guess.
c. Reasonable person standard is different in an emergency
d. Also, judge tells us that people should not be expected to be super-heroes. Defendant was watching out for
his own skin and this was justified.
e. Judge does not want to pass judgment in this situation.
f. Cordas is like Vincent v. Lake Erie It is like not picking a dock. Maybe if ship-owner did not pick a dock he
would not be liable.
g. Special jury instruction People argue this, but it should not happen because it is implicit in the first
instruction that you should take circumstances into consideration.
i. NY is a special instruction state. Might make it unbalanced but ensures that juries do it.

8.

Industry Custom
a. When you allow custom evidence into negligence cases it determines what the standard of care is, and jury
does not determine the standard. It tells jury the answer to the negligence question. It is let in because it is
just so important.
i. Custom tells the BPL B not too high because other people are able to do it and everybody knows
the danger.
b. Unlikely that the jury will find in favor of a D who has violated industry customs
c. Can conform to industry customs and still be held liable
d. Violation of custom is not binding, it is relevant.
i. The only way to beat custom is to argue that the custom speaks to a practice that is not directly on
point with the case at bar.
e. Conformance to custom is not binding, it is relevant. Jury can find the whole industry negligent. Need very
good evidence to show a whole industry is negligent and get passed directed verdict to get to the jury.
i. TJ Hooper Case JURY MUST STILL DETERMINE WHETHER WHAT WAS DONE,
CUSTOMARY OR NOT, COMPORTS WITH THE NEGLIGENCE STANDARD ITSELF
ORDINARY CARE UNDER THE CIRCUMSTANCES.
It was customary for tug owners not to equip tugs with radio equipment. Two barges were lost at sea
because they did not have radios and could not be informed of coastal storm. Still held liable.
1. Judge Learned Hand Even their universal regard will not excuse their omission.
f. Now and then courts direct a verdict on custom. Says this industry is not crazy.
g. Medical custom is relevant and BINDING.

9.

Trimarco v. Klein Tenent sues his landlord for negligence for not using shatterproof glass in bathroom.
a. Expert testimony showed the common use of shatterproof glass.

13

b.

i. Expert testimony shows that the industry has already done the risk utility analysis burden is not too
high.
Twerski has a number of problems with this case:
i. Custom (which is very persuasive) does not speak to the facts of this case. Custom speaks to
replacement and refurbishment.
1. If you let custom in, it ought to be on all fours.
ii. If custom did not come into the case, plaintiff could still argue:
1. costs of shatterproof glass low
2. widely used
3. and injury severe.

10. Can your own custom be used against you? HYPO 20 Handbook provides that suspected shoplifters should not be
detained for longer than 15 minutes. Someone suspected of shoplifting was detained by a security guard for 30
minutes for investigation. Statute provides that suspected shoplifter may be detained for a reasonable amount of
time.
a. Twerski does not know the answer.
b. The handbook is not totally irrelevant, because if you set a rule it shows what you think to be reasonable.
c. But you could set a very high standard to avoid the exact trouble that you just got in to.
d. Better argument for this kind of custom to stay out of evidence and let the jury decide.
11. Roberts v. State of Louisiana No different standard of care for handicapped. Might have to be more careful but
they do not have to be more reasonable, just reasonable under the circumstances.
Blind man left his post at a concession stand to make a familiar walk to the bathroom without a cane, and hit plaintiff
who fell to the floor.
a. Physical and Mental Attributes Do we hold handicapped person to a higher standard of care? NO. Standard
of care of a reasonable person.
b. Do factors of BPL change? YES. But only one standard of reasonableness (the weighing process) and it is
the same.
c. Custom most people do not use a cane. It is accepted even though it is dangerous this is somewhat
scary
d. Court directed the verdict set the standard of care did not set a rule for handicapped people. Just must act
as a reasonable person under the circumstances
e. Twerski says case is close to going to a jury but jury could be inconsistent and then the blind would have to
use a cane to prevent against liability.
12. Mentally Ill Courts do not get into the stages of mental illness. Do not want to fuss with it as a defense. Reasonable
person standard is semi-objective this cannot work with mentally ill because mental infirmities are invisible, hard to
measure, and incompletely verifiable.
a. Policy decision- no real break for mental incapacity
i. Want to tell people who take care of mentally ill to take care of them.
ii. Poor mentally ill will probably not get sued.
13. KIDS Only place where BPL Formula is changed. Age, Experience, Intelligence Test (AEI)
a. Restatement Children:
i. When the actor is a child, the actors conduct is negligent if it does not conform to that of a
reasonably careful person of the same age, intelligence, and experience; except that
ii. A child who is less than five years of age is incapable of negligence; and
iii. The special rule does not apply when the child is engaging in a dangerous activity that is
characteristically undertaken by adults- (example: driving)
b. Special rule for kids because:
i. Dont have good data
ii. Cannot tell them Should have known because it takes years. Cannot improve data at the current
time. Also cannot keep kids out of harms way because it is part of the maturation process.
iii. EXCEPTION when kids are engaged in an adult activity.

14

14. Stevens v. Veenstra Veenstra, 14-year-old, taking a drivers education course and hits gas instead of break while
turning too sharply and hit plaintiff.
a. Defendants argument: Im a kid in a state mandated program. Twerski says nice but it did not work.
b. Twerski says we should blame the instructor
c. What if adult was driving for the first time? Sense is that we do not make allowances for beginners (Beginner
surgeon).
d. What about when young kid takes keys to parents car without permission and drives? Probably held to a kid
standard because doing a very childlike thing. Should-have-known is a dead give-away.
e. What is an adult activity? Firearms, driving snowmobile, operating motor scooter and golfing have gone both
ways in different jurisdictions.
15. Is there a special standard of care for professionals?
a. Restatement:
i. Unless he represents that he has greater or less skill or knowledge, he who undertakes to render
services in the practice of a profession or trade is required to exercise the skill and knowledge
normally possessed by members of that profession or trade in good standing in similar communities.
b. No different rules, just articulated because the question is Did you act reasonably under the circumstances,
meaning within the profession?
i. Expert status is a circumstance that colors the meaning of reasonableness.
ii. Unless you make less skill/knowledge known to patient, then if you offer a service you must have the
normal skill and knowledge.
iii. Professional has an obligation to acquire knowledge should have known is high.
c. In medical malpractice cases, adherence to custom gets D off.
16. Boyce v. Brown NEED TO ESTABLISH PROFESSIONAL NEGLIGENCE THROUGH EXPERT KNOWLEDGE
OF CUSTOM.
Doctor put screws in defendants ankle, No problem, later, patient complains of pain, Doctor looks at ankle and wraps
it. Patient goes to another doctor who does an X-Ray, finds necrosis of the bone around the screw, removes the screw
and the ankle heals.
a. Plaintiff cannot make out a case because expert testimony does not speak to custom. Does not say X-Ray
was custom. Instead says, I would have given X-Ray not enough.
b. Medical custom is relevant AND binding.
c. What if he had taken the X-Ray? Still problem of PROXIMATE CAUSE. What would the x-ray have
shown?
d. Rules of law governing actions of malpractice
i. one licensed to practice medicine is expected to have degree of skill and learning possessed by
average member of medical profession- if not, is guilty of malpractice
ii. to be held liable for malpractice, must have done something in his treatment of patient which the
recognized standard of good medical practice in the community forbids in such cases, or neglected to
do something which standard requires
iii. in order to sustain verdict for plaintiff, the standard of medical practice in the community must be
shown by affirmative evidence
iv. negligence is never presumed, but must be affirmatively proven. Can be no presumption of
negligence just b/c treatment didnt work
v. negligence must be established by expert medical testimony, unless the negligence is so grossly a
pparent that a layman would have no difficulty in recognizing it
vi. testimony of other physicians that they would have proceeded differently is not sufficient to est
malpractice unless it also appears that course of treatment deviated from one of the methods of
treatment approved by the standard in that community
17. JUDICIALLY DETERMINED STANDARDS OF CARE: court can set the standard when they believe that reasonable
persons cannot differ but decisions 4-3 are close. Basically not ready to impose burden on defendant so jury cannot
make a decision. Very fact sensitive

15


1.
2.

In most negligence cases, juries perform 2 functions:


resolve conflicts b/t parties as to what occurred
set the standard of reasonable care under the circumstances

Helling v. Carey Doctors fail to do a pressure test on Helling to detect for glaucoma because the profession does not
require the test for patients under 40 because the incidence is one in 25,000 persons under the age of 40. By the time the
glaucoma is detected the damage on her vision is extensive.
a. Directed verdict for the plaintiff did not go to jury.
b. Judges were ready to take on medical custom and impose liability.
c. Mystery as to what this case actually holds:
i. Says under the facts of this case but Hellings chances for glaucoma are much higher because she
went to the doctor so many times.
ii. Is this limited to glaucoma testing?
iii. Is this limited to any medical risk of one in 25,000?
iv. Slippery slope- this is a non-evasive tests, but there are many non-evasive tests that arent done on
people
18. Violation of a statute
a. Restatement Gary Schwartz. An actor is negligent if, without excuse, if
1. the actor violates a statute
2. that is designed to protect against the type of accident the actors conduct causes, and,
3. if the accident victim is within the class of persons the statute is designed to protect.
ii. Reasoning: We take statutory standards seriously because:
1. the legislature has already said what the minimum standard is
2. Negligence is hard to administer we live with inconsistency buy why live with it when
the legislature has given a clear standard?
b. When statute is admitted
i. The legislature has already set its opinion the jury should have less discretion.
ii. Jury decides the fact and if causal relationship exists.
c. Cost of using a statute statute may not provide a good fit~
i. Statutes are written in global terms, Thou shalt never and there are always exceptions and that is
a minimum standard.
ii. Not written for tort cases which are fact sensitive.
iii. Takes away the violins. Makes you negligent when a jury could find for you, or, even more
powerful, a judge could direct a verdict for a defendant.
iv. Statute is written for something else (Driving- get a small fine) and not for torts (Damages could be
millions of dollars). Statutes not written for million dollar lawsuits.
19. How different states treat violation of a statute?
a. Negligence per se The judge decides if the statute is applicable (sometimes the statute is inappropriate
considering the circumstances) and then directs a verdict on the standard of care. Jury only decides:
i. Did actor violate statute
ii. Causal relationship
b. what violation of the statute actually means:
i. majority follows Martin- violation of statute= negligence per se
1. judge decides if the statute is applicable and directs a verdict on standard of care
2. jury decides if actor violated that standard, and whether there was a causal connection b/t
the negligence and the harm
ii. some states treat statutory violation as some evidence of negligence
1. jury hears the allegedly negligence conduct and takes the statute into account in deciding what
the standard of care should be
a. treats violation of statute like custom
iii. handful of states say that violation of statute creates a prima facie case or presumption of negligence
1. can be rebutted by proof that a reasonable person would have acted as did the person whose
conduct is in question- jury decides reasonableness of conduct, statute doesnt play significant role

16

c. Restatement- an actor is negligent if, w/o excuse, the actor violates a statute that is designed to protect against
the type of accident the actors conduct causes, and if the accident victim is within the class of persons the statute is
designed to protect
c. Negligence per se with Excuse Most common + Restatement
i. It is negligence per se but can offer justification. Restatement has 5:
1. incapacity
2. lack of knowledge of problem (dont know headlight is out)
3. inability to comply (blizzard)
4. emergency
5. compliance poses greater risk than violation
d. Violation of Statute is Some evidence of negligence Jury can consider violation but not compelled to find
negligence even in the absence of rebutting evidence.
i. Similar to custom, relevant but not binding.
ii. This makes the jury take on the role of the legislature.
e. Presumption of negligence presumption of negligence that shifts the tennis ball to the other court and can
show that reasonable person would have acted as he did. Jury decides reasonableness of actors conduct and
statute does not play a role.
20. Martin v. Hertzog IF JURY FOUND VIOLATED STATUTE THAN JURY WOULD HAVE TO FIND
NEGLIGENCE AND CANNOT LOOK AT ANY EXCUSES.
Plaintiff was driving in buggy without his lights on when he was hit by an automobile tat was driving around a bend,
driving in the middle of the road. said couldnt see the buggy. There is a statute that said had to have lights on.
a. Plaintiff asked that statute be used as evidence of negligence.
b. Defendant asked for prima facie negligence
i. a presumption of negligence which shifts the burden to the other side. Forces the other side to say
something to show that he acted reasonably. If other side is silent there will be a directed verdict
against him.
ii. Cardoza did not like this because the other side would use violins.
c. Trial judge told the jury to take the statute into account to throw it into the hopper! This is treating it as
custom and that is erroneous.
d. Cardoza said failure to comply with statute is negligence per se
i. Negligence due to the violation of a public duty, such as speeding.
e. Said this case is below 2 and you are not behaving no excuses. There was a standard of care and you
violated it.
f. Most courts have softened this strict/rigid stance.
21. Reque v. Milwaukee & Suburban Transportation Corp THE STATUTE MUST ESTABLISH A RELEVANT
DUTY OF CARE.
Plaintiff fell getting off a bus and contended bus violated Wisconsin statute that required parked vehicles to be 12
inches from curb.
a. Court did not apply statute because intent of statute did not have anything to do with people getting off a bus.
i. The was not a member of the protected party that was the intent of the statute.
ii. Purpose was to get busses close to curb so they wouldnt get in the way of traffic
b. Most courts only use statute when it is relevant in establishing negligence if it is meant to protect the persons
like the plaintiff from the type of harm which actually occurs.
Can a statute with one purpose take on a secondary meaning?
22. Ney v. Yellow Cab Co. COURTS WILL REFUSE TO TREAT VIOLATION OF A STATUTE AS NEGLIGENCE IF
THE STATUTE WAS NOT INTENDED TO PROTECT THE CLASS OF PERSONS TO WHICH THE PLAINTIFF
BELONGS.
a. Statute requires that a driver must turn off car, lock the ignition and remote the key, before getting out of the
car.
b. What is the intent:
i. Protect citizens from the negligent driving of a getaway thief
ii. Prevent inadvertent or negligent movement of parked vehicle.
c. Court decides it should be a public safety statute.

17

d.
e.

Cabbies commonly leave keys in car. If someone steals the car and hits the pedestrian, is the cabbie liable to
the pedestrian?
Factual variations create a jury issue
i. This makes a situation where the statute governs but the jury decides if it protects a particular class of
people. Twerski says this is a decision the judge has to make!!!
ii. The court doesnt know (usually decide if defendant is negligent per se) if the statute should apply, so
how should the jury know?

23. Stachniewicz v. Mar Cam Corp. JUDGE WILL DETERMINE IF A STATUTE IS APPROPRIATE FOR A TORT
CASE.
Defendant gets injured in a bar brawl and sues bartender for serving someone who was already intoxicated.
a. Judge HOLMAN had 2 sources of authority:
i. Statute Thou shalt not serve someone who is already drunk
1. Holman does not use this standard because it would be possible to determine CAUSATION.
What is the difference between drunk and super drunk? Is this the difference
a. Twerski thinks this is wrong. Should be able to sue guy who drank and drove.
There is a way to deal with causation.
ii. Regulation: Keep the bar quiet, dont let people get out of hand.
1. Holman applies this standard of care.
2. The regulation seeks to prevent abusive conduct and drunken clientele which results in
serious injury to customers.
3. The bar owners breach of duty to protect his duties from harm results in drunken brawl and
that is negligence
4. Plaintiff was within class of persons intended to be protected by the regulation and harm
caused to him was type intended to be prevented by statute.
b. Holman says Dont Ask Me to be a Robot. I will tell you whether I think a statute is appropriate for a tort
case.
24. Perry v. S.N. STATUTE COULD NOT BE USED TO MAKE A DUTY TO REPORT SEXUAL ABUSE.
Statute says a person must report any person who is sexually abusing children. Plaintiff brought a suit against wife
because she should have known that husband was sexually abusing teenage girl.
a. Court decided criminal statute should not be imported into a civil negligence case.
b. Tort law does not impose a duty.
c. Amorphous Standard Statute says you act when you have a cause to believe did not set the standard of
care.
25. Licensing Statutes are not negligence per se cases... they tell you the skills you must have, and only by proving that
you dont have those skills, not be proving that your dont have the license can you be negligent
a. License says you should not be doing what you are doing AT ALL. Therefore the defendant has to come up
with proof that he was acting reasonably, and then we will use the reasonable person standard.
b. Different from Martin v. Hertzog where it is negligence per se because statute set the standard of care and
defendant violated it.
c. Brown v. Shyne chiropractor did not have license but not negligence per se. If defendant could introduce
info that he had requisite knowledge and skill to procure license then throw statute out completely. Must
look at injury was due to unskilled or carelessness.
d. A nurse without a license performs surgery not automatic negligence.
e. Should violation of a licensing statute be a directed verdict for negligence? Statute does not set standard of
care and therefore does not speak to negligence.
26. Ipson v. Structural Metals, Inc. NEGLIGENCE PER SE EASES THE BURDEN OF PROOF FOR THE
PLAINTIFF BUT STATUES DO NOT SPEAK TO SPECIFIC SITUATIONS.
Truck tried to pass on left while car made a left turn.
a. Statutue: Thou shalt not pass within 100 feet of intersection.
b. Defendant knew about the intersection and he defends himself by saying, I was not negligent, I made a
reasonable mistake under the circumstances.

18

c.

Court says D is liable b/c Judgment of Court of Civil Appeals is reversed, and the judgment of the trial court is
affirmed.
d. NOT a defense to say If statute was not there I would be acting reasonable. That is the exact problem of
using the statute.
e. There are NO VIOLINS in a negligence per se case.
i. A justification could be something like an emergency.
ii. Statutes do not speak to specific situations, so at some point violins are needed.
Notes: Restatement of Torts- excused violation of a legislative enactment is not negligence, lists 5 categories (not exclusive)
1.
violation is reasonable b/c of actors incapacity
2.
he neither knows nor should know of the occasion for compliance
3.
he is unable after reasonable diligence or care to comply
4.
he is confronted by an emergency not due to his own misconduct
5.
compliance would involve a greater risk of harm to the actor or to others
- it is not an excuse to say I acted reasonably even though I violated the statute
27. Res Ipsa The thing speaks for itself. Allow jury to draw inference from common sense inferences.
a. - we admit that we dont know what went wrong, but are willing to conclude that whatever went wrong was
more likely than not the result of negligent conduct
b. - requires that the instrumentality or agent that caused the accident had been under the exclusive control of
the defendant
c.

d.

e.
f.

Rules:
i. The accident ordinarily does not happen unless someone was negligent.
ii. The instrumentality or agent which caused the accident was under the exclusive control of the
defendant.
1. Twerski said maker of this rule should be shot at sunrise. No problem finding the donkey,
problem pinning the tail on the donkey.
iii. The circumstances indicated that the untoward event was not caused or contributed to by any act or
neglect of the injured party.
General statements:
i. I dont know what went wrong, but when I try to think of all the things that could have gone wrong,
negligence is the most probable explanation.
ii. I cannot tell you what the negligence is.
iii. More likely than not, you are the guy that caused the negligence.
iv. NO evidence, using generalizations.
Most successful way to beat res ipsa is to put doubt that generalization is working. Put butterflies in judges
stomach. Show that there are non-negligent alternatives that give a reasonable explanation of what happened.
Bryne v. Boadle Barrel of flower fell out window. Mere fact of accident having occurred is evidence of
negligence.

28. Ybarra v. Spangard After appendicitis operation, defendant suffered pain around neck and back.
a. Problem do not know who caused the damage? P was under anesthesia and many people in operating room.
b. Court tells each defendant to prove why he is not guilty shifts burden of who comes forward with evidence.
c. Every defendant says, Not me, I didnt come near neck. Impossible to determine who the negligent person
is and cannot impose liability on anybody.
d. Somberg v. Anderson Tip of tool fell off and lodged in plaintiffs back during surgery.
i. Possible defendants:
1. Hospital in maintenance of tool
2. Manufacturer of tool
3. Surgeon
4. Distributor of instrument
ii. Court said it was not sufficient to shift the burden of proof. Instructed jury that they must come in
against the most likely defendant.

19

29. Sullivan v. Crabtree RES IPSA EFFECT AT TRIAL DIFFERENCE BETWEEN INFERENCE AND
PRESUMPTION.- res ipsa doesnt demand finding of negligence in every jurisdiction
Plaintiff sues for death of son, who was killed while a passenger in a truck that went off a steep embankment.
a. Res ipsa instruction was given to the jury.
b. On appeal, plaintiff asks for a presumption of negligence and not just an inference of negligence.
i. Presumption (prima facie case) would require the defendant to present evidence that rebuts the
presumption.
1. Same with statutes.
c. Defendant won the case because it was just an inference (merely provides evidence to support inference that
defendant was negligent). He says he does not know what happened to make truck go off embankment. He
did not say enough to rebut a presumption.
d. Jury found for defendant because they believe the experienced truck driver knew something about driving on
a curvy road.
e. Plaintiff was trying to change the law. Trying to make Tennessee one of the states that treats res ipsa as a
presumption and not just an inference.
f. Some cases you can treat it as an inference and you are going to get a directed verdict in favor of the plaintiff:
i. Mouse in coca cola bottle.
ii. Only defense is to say it never happened. Credibility question.
30. Stelter v. Chiquita safety inspector fell through safety grate. Only Chiquita employees remove and replace the
grates. P alleges negligence and asks for res ipsa loquitor instruction
a. This case satisfies the 3 elements necessary for a res ipsa loquitur instruction.
i. this type of occurrence doesnt happen without negligence
ii. instrumentality that caused injury was within Ds exclusive control
iii. injury wasnt caused by P
CHAPTER 4:
ACTUAL CAUSATION
1.

Theory Underlying Causation:


a. A plaintiff can prove negligence, but if no causation is proved, no recovery.
b. Had D performed differently would harm still have occurred?
c. Cannot hold liable for the risk because:
i. Cannot assess damages
ii. People are not injured, no need for money.
iii. Impossible to operate.
d. We impose liability for practical reasons, not deep principle
i. Twerskis theory that informed consent.
e. 2 causation questions:
i. was it the conduct of this D that caused the harm and
ii. was Ds negligence the but-for cause of the harm

2.

Perkins v. Texas and New Orleans Ry. YOU ARE NOT THE BUT FOR CAUSE IF IT IS A MATTER OF
TIMING.
Speeding train hit car in blind-intersection.
a. In this case the court used the internal custom against the party who created the custom.
b. The negligence is the 12MPH over the speed limit the train was traveling. Question: If train had not been
going 12 MPH over the speed limit would the accident not have happened? Admit that defendants conduct
had something to do with harm. But still need the negligence to make the difference.
c. No, it appears almost certain that the fatal accident would have occurred irrespective of the excessive speed of
the train- therefore speeding was not the cause of the harm
d. This is called a hypothetical but for because we have no way of knowing what would have happened but
for the defendants negligence.
e. Cannot prove escape theory that plaintiff would have had more time.
f. The negligence has to LEGALLY be the cause of the accident (not for jury).

20

3.

Ford v. Trident Fisheries Co. DIRECTED VERDICT BECAUSE EVEN IF PRECAUTIONS HAD BEEN MADE,
THERE WAS NO CHANCE OF SAVING THE GUY.
Man fell overboard and it took longer to get boat in water and they only had one oar.
a. Cannot get past directed verdict, so could not get violins.
b. The guy had already sunk before, so could not prove that defendant did anything that contributed to his death.
c. Are we protecting the CHANCE? If so, then this case might have gone to a jury.
i. Authors Dialogue:
1. Twerski he would have died no matter what, dont want to deter people from engaging in
dangerous activity.
2. Jim this verdict is wrong. Need to encourage people to take reasonable care.

4.

Lyons v. Midnight Sun Transportation Services, Inc. PLAINTIFF CANNOT JUST PROVE NEGLIGENCE,
MUST PROVE THAT THE NEGLIGENCE WAS THE CAUSE IN FACT OF THE ACCIDENT.
Car was speeding, but the speeding did not cause the accident.

5.

Gaps in Proof Would a sign that read No lifeguard on Duty really make a difference? Probably not but we hold
the hotel liable anyway.
a. Ways to avoid:
i. Prove plaintiff knew of the harm.
ii. Prove plaintiff would not have headed the warning. (Did not read instruction manual).
b. TWERSKI- recommends a presumption of causation in failure to warn cases.

6.

Reynolds v. Texas Pacific Ry. - WHERE COURT WANTS POLICY, AND CLASS OF DEFENDANTS NEED
PROTECTION, AND PLAINTIFF HAD TO KNOW BETTER COURT SMUDGE CAUSATION AND RULE
ACCORDINGLY.
Fat Mrs. Reynolds falls down dark staircase without banister.
a. Courts will smudge the causation issue in order to make good policy. Cannot prove that she would not have
fallen even if there were adequate precautions taken.
b. No causation needed when the negligence of the defendant generally (greatly) multiplies the chances of the
accident.
c. Where negligence of D greatly multiplies the chances of accident to the P, and is of a character naturally
leading to its occurrence, the mere possibility that it might have happened without the negligence is not
enough to break the chain of cause and effect between the negligence and the injury.
i. Counter-argument: In every negligence case the risk is increased.

Failure to Warn and Actual Causation


if the risks of harm are generally obvious to reasonable people, no duty to warn arises
when the risks of harm are not generally obvious, but the D proves that the particular P knew of the danger
from other sources, courts have ruled as a matter of law that the Ds failure to warn didnt cause the accident
even when the P doesnt know of the relevant risk, the negligent D will escape liability by proving that the P
wouldnt have read and heeded a warning, had one been given
7.

Kramer Service, Inc. v. Wilkins RULE: COINCIDENCE DOES NOT EQUAL CAUSATION. PROBABILITIES ARE NOT SUFFICIENT TO MAKE
OUT CAUSATION.
Plaintiff hit with glass and spot became cancer.
a. Question: Is there enough of a policy concern to warrant causation and put the onus on the defendant?
b. Defendant argues glass does not cause cancer nothing in my conduct implicates me stronger argument
than I was not negligent.
c. Medical expert says 1 in 100 chance of causation need MUCH more than that.
d. When medical experts are only advisory and the harm is beyond the range of knowledge of experts, then there
cannot be recovery. To make a case you need the undisputed testimony of reputable specialists.
e. PROBABILITIES ALONE ARE NOT ENOUGH

21

f.

MEDICAL TESTIMONY MUST SHOW A REASONABLE PROBABILITY OF CAUSATION

8.

Daubert v. Merrell Dow Pharmaceuticals, Inc. RULE: JUDGES DETERMINE THE RELIABILITY OF THE EXPERT TESTIMONY AND JURIES DECIDE THE
PERSUASIVENESS.
Birth defects due to Bendectin anti-nausea drug and plaintiff had controversial expert studies that attempted to
show causation.
a. High threshold must prove that medicine more probably than not caused the harm (doubled the risk).
Defendant is going to say the harm was a background risk.
i. General acceptance in medical community isnt absolute prerequisite to admissibility
b. Frye crap medicine got into the court and the defendant had the burden of proving it wrong.
c. After Daubert Judge has the power to weed out the plaintiffs only proof of causation.
d. Made federal courts gate keepers between good science and garbage.
e. Who is better to weed out bad medicine information? Judge or jury? Daubert says judge, implies reasonable
person cannot make sense of highly technical information.
f. Four facts for when something is admissible:
i. Can the theory be tested?
ii. Has it been subject to peer review?
iii. What is the rate of error of scientific protocol that you are using?
iv. Then look at general acceptability.
g. How can judges determine good science? With cutting edge medicine doctors and scientists do not always
agree.
h. Case sent back down to 9th circuit criticizes expert testimony and affirms summary judgment for plaintiffs
because:
i. Plaintiffs studies done by hired guns
1. Counterargument plaintiff always has to hire doctor.
ii. Issue of peer review (Dauberts studies published and not peer reviewed)?
i. CAUSATION Injury Causation is an issue. Have a violation of right (inform) and no injury causation so the
law does not provde a remedy.
j. Twerski: Daubert negates a substantial tort right that should be protected. There is sufficient evidence that she
was not informed of a risk (even though the risk was un-definable) and this gives her a perfectly good cause of
action.
i. Twerskis right to informed choice is there even without injury.
ii. If this was handled as informed consent no woman would take the drug. Problem is that there was no
harm. What about the mere fact that you took drug when you didnt know of risks.
k. Case represents a substantive decision not to recognize that there is an informational tort that should be in the
hands of consumers and not in the hands of the company.
l. HUGE implications for pharmaceutical companies because cases get directed verdicts and dont make it to a
jury.

9.

Rider v. Sandoz Pharmaceutival Corp.- Ps took drug to suppress lactation, both subsequently suffered hemorrhagic
stoke
a. studies arent strong enough to demonstrate adequate relationship b/t drug and strokes
b. FDA taking the drug off the market is not enough to prove causation

10. Herskovits v. Group Health Cooperative of Puget Sound PROTECT A SIGNIFICANT LOSS OF CHANCE (OR SIGNIFICANT INCREASE IN HARM)= SUFFICIENT
CAUSATION.
Failure to diagnose cancer.
a. Defendant is held liable for the reduction in defendants chances of surviving.
b. This case really had numbers for the first time, would have failed in Ford v. Trident.
c. The harm is the CHANCE and not the death.
i. Old rule: More likely than not cause of death.
ii. New Rule: More likely than not he would have had a chance.
d. When you have a significant loss of chance, court will send it to the jury and they can take the loss of chance
into account and figure out the damages as they see fit.

22

e.
f.
g.
h.
i.
j.
k.
l.

Once jury finds percentage of chance diminished, that is percentage of damages paid.
The patients possibility of 5-year survival was 39% and diagnosed 6 months later and his possibility of
survival was reduced to 25%. The delay in diagnosis allowed the tumor to progress from stage 1 to stage 2
and reduce 14% enough for jury to decide.
Once you do proportional recovery you have to be consistent. Now, defendants can argue I just diminished
the chance so I will pay the percentage. Some plaintiffs lose and lose plaintiffs win but you cannot have it
both ways.
Many courts have stuck with old rule more probably than not to avoid problems.
But For is the same as more likely than not. Herskowitz sharply cuts into tort law.
Overcompensates in cases where decedent would have died anyway.
Under- compensates in cases where delayed diagnosis caused death.
When the epidemiological info is not available courts are uncomfortable allowing recovery.

When Two (Or More) Negligent Actors Concurrently (or Successively) Cause the Ps Harm
11. PROBLEMATIC HYPOS.
a. Restatement actors negligent conduct is a legal cause of harm to another if his conduct is a substantial
factor in bringing out the harm. - cant really use traditional but-for test
b. Fire Hypos:
i. 2 negligent fires burn a house at the same time neither but for cause both held liable.
1. Both say, but for my negligence the other fire would burn.
2. Joint and Several Liability does not work Restatement says substantial factor.
ii. 1 negligent fire + 1 unknown fire neither but for cause negligent held liable.
iii. 1 fire, then another fire neither but for cause first fire responsible, second fire is not.
1. 1st fire physically did the damage
2. 2nd fire lucky enough to risk the damage, but not the cause of the harm.
c. You are usually now just responsible for your percentage of the fault
i. At common law, a tortfeasor could be held liable for the whole burden, most states have statutes
which proportions among joint tortfeasors now
12. Hill v. Edmonds RULE: YOU ARE STILL LIABLE EVEN IF YOU CAN POINT TO SOMEONE ELSES NEGLIGENCE.
Tractor in middle of road with no lights hit by car who should have been paying attention
a. 2 but for causes So neither was the but for cause, each was a substantial factor, both are liable.
i. Law chooses over-deterrence rather than under-deterrence.
b. Where separate acts of negligence combine to produce a single injury, each tort-feasor is responsible for the
entire result, even though his act alone might not have caused the accident.
c. It is no answer for your but-for negligence that someone else was also negligent
i. You dont get off because somebody else was also negligent
13. Kingston v. Chicago & N.W.
Rule: IF YOU ARE A SUBSTANTIAL FACTOR YOU ARE LIABLE (DO NOT NEED TO BE THE BUT FOR
CAUSE).
a. One fire is negligently set and the other is of unknown origin. Negligent fire liable.
b. Neither fire is really the but-for cause. Either could have burnt the house without the other. Use substantial
factor test for liability.
14. Summers v Tice RULE: IF DONT KNOW WHICH CAUSED HARM, COURT SHIFTS THE BURDEN TO THE DEFENDANTS.
2 hunters shot, dont know which hit eye and which missed.
a. Neither substantial. Neither more probably than not the cause of harm.
b. Shows how sometimes applying the but for test exonerates a negligent defendant.
c. Unless defendant can prove himself out, then he is negligent.
d. Tort law does not believe you should punish for taking a risk, punish for taking a risk PLUS causing harm.
However, in situations where there is no way of differentiating it is unfair to deny the plaintiff recovery.

23

15. Sindell v. Abbott Laboratories


RULE: Once it is clear that some of the defendants injured some of the population, the defendants will not escape
liability merely because the plaintiffs cannot show which defendants injured which plaintiffs.
DES cases.
a. Have negligence, have duty, have causation, and have harm. Dont have defendant!
b. Court rejected Summers v. Tice. Too many defendants, and dont even have all the defendants that caused the
harm. If you shift burden then those defendants who could not prove their innocence would be held liable for
100% of the harm when maybe they only caused 5% (or no harm at all).
c. Market share liability Each defendant pays for the harm he caused to the world.
d. The wrong is corrected, but not necessarily by the party who wronged the plaintiff.
e. NY Court of Appeals - once we do market share we acknowledge that we cannot figure out defendant so we
will not litigate individual cases.
f. Twerski if you use Market Share liability you cannot impose joint and several liability.
g. Proportional causations:
i. Herskowits I can prove defendant but not harm.
ii. Sindell I can prove harm and not defendant
h. Leave traditional mode and move into proportionalization.
CHAPTER 5
PROXIMATE CAUSATION
a.

b.
c.

2.

Look at the composite of the risks (foreseeability/scope-of-the-risk list)


i. If defendant should have anticipated a particular risk at the time he acted, and he negligently
failed to avert that risk, he would be liable if that risk caused the plaintiffs harm.
ii. If plaintiffs injury is truly beyond the type of harm to be expected the plaintiff will virtually
always go uncompensated.
What would the reasonable person do? Could reasonable person see that risk and avoid it?
Was the harm on the risk list? Was the risk that materialized the principal risk that made the defendants
actions negligent?
i. The sequence of events might be bizarre (rat) but if the nature of the harm was foreseeable than
still liability.
ii. Was it in the bundle of risks the riskier your conduct the longer the list. Intentional tort or tort
feasor than intent is proximate cause.
iii. If defendant is negligent, and anyone could see that such harm could occur, the manner in which
the harm occurs (be it random, freakish, even unforeseeable) is still considered proximate cause.

Basics:
a. Foreseeability
i. Unforeseeable plaintiffs (Palsgraf)
ii. Unforeseeable types of harm (Wagon Mound)
iii. Unforeseeable extent of harm (thin-skull)
iv. Unforeseeable manner of harm (Marshall v. Nugent)

LIABILITY LIMITED TO REASONABLY FORESEEABLE CONSEQUENCES


3.

Marshall v. Nugent RULE: LIABILITY FROM THE OPERATION OF RISK. ISSUE FOR JURY
Defendants driving caused car to run off harm (defendant unharmed.) Then defendant was hit by another car
when walking down road.
a. Probability of accident was low, but you have to look at the composite of the risks- BUNDLE OF RISKSis the risk of getting hit by another car part of the bundle of risks?
b. When you are negligent, you are liable not just for the foreseeable risks you create, but for all the risks
that could occur as a result
c. Driver of truck tied up traffic in a dangerous blind spot.
d. Jury decides unless judge wants to direct a verdict.

24

e.

TWERSKI LIKES THIS

4.

McCahill v. New York Transporting Co. (1911)


TAKE YOUR PLAINTIFF AS YOU FIND HIM- THIN-SKULL RULE
Cab driver hit an alcoholic.
a. If you are negligent in the first place than you can be held liable for a consequence that could have
occurred without your negligence (man could have had delirium tremors from alcohol, but accident
induced them).
TWERSKI SAYS:
you have to assume you will hit a guy with a thin-skull and this is how you set your standard of care
you cant assume when setting your standard of care that people have an ordinary capacity to resist injury
THIN-SKULL RULE
one who has negligently forwarded a diseased condition and thereby hastened and prematurely caused death
cant escape responsibility even though the disease probably would have resulted in death cannot escape
responsibility even though the disease would have resulted in death at a later time without his agency
o might have to pay less damages b/c shorter life expectancy anyway
5.

Wagon Mound #1
RULE: IN RESPECTS TO PROPERTY, HARM MUST BE FORESEEABLE FOR RECOVERY.
Ship discharges oil, people working on wharf using torches fire.
a. Same as Polemis thin skull rule as to property boards fell and ship blew up.
b. Court rejects Polemis. If not foreseeable, than cannot be contributed to negligence. Only liable for
foreseeable consequences of releasing the oil, not for unforeseeable fire which destroyed the dock itself.
c. Risk of fire was so remote it shouldnt have been considered
d. Total bar for contributory negligence, so plaintiff did not push foreseeability issue because did not want to
be found at fault.
e. If the risk was foreseeable in this case, then P was contributorily negligent- knew the oil was in thewater

6.

Wagon Mound #2 RULE: LIABILITY AWARDED FOR DAMAGES TO BOATS FROM FIRE.
Ship-owners sue no problem of contributory negligence so scream foreseeability.
a. Analysis terrible attaches social value analysis which does not answer the scope of risk question.
b. What happened to bundle of risks?
c. Cannot do risk-utility balancing for very remote risks because we do not live our lives worrying about
remote risks.

7.

Palsgraff v. Long Island Rail Road THE ACCIDENT IS OFF THE RISK LIST SO WE CALL IT NO DUTY.
Dynamite hits track, explosion, scales fall, Mrs. Palsgraf is hit and sued.
a. Foreseeability determines scope of the risk.
b. The employers were evidently negligent but their negligence only posed a foreseeable risk of injury to the
passenger or his package, not to Mrs. Palsgraf.
c. Cardoza uses the word Duty to signal a judge to direct a verdict.
d. The risk reasonably to be perceived defines the duty to be obeyed
i. Negligence is a matter of relationship between the parties
ii. Relationship must be based upon the foreseeability of harm to the person who is, in fact,
injured- this risk wasnt in the bundle
iii. Just b/c the Ds act was negligent, it is not necessarily wrong to the injured party
iv. If there is no duty to the injured party, cant be held liable for injury done to him
e. TWERSKI doesnt really agree- what about the poor P?
f. DISSENT Andrews:
i. Policy talk. If you get past foreseeability you might want to look at the case as a matter of good
public policy.
ii. Do you want to hold someone liable for the acts of an arsonists.
iii. We will cut off liability but we will not use foreseeability to decide when.

25

iv. Just because something is unforeseeable does not take it off the risk list.
v. Accepts that judges could cut of liability arbitrarily breakfast bad.
8.

Deridiarian v. Felix Contracting Corporation. p. 261


IF A SUPERSEDING CAUSE IS FORESEEABLE THERE IS LIABILITY.
Driver did not take medicine, crashed into construction site, worker landed in boiling enamel. Felix did not
construct an adequate barrier- D found liable, at least for a portion of fault
a. There are intervening causes that are unforeseeable and foreseeable. If it is foreseeable than there could
be liability.
b. Could do contributory negligence.
c. Where the acts of a 3rd party intervene b/t the Ds conduct and the Ps injury, the causal connection is not
automatically severed. In such a case, liability turns on whether the intervening act is a normal or
foreseeable consequence of the situation created by the Ds negligence.
d. If the intervening act is extraordinary under the circumstances, not foreseeable in the normal course of
events, or independent of, or far removed from the Ds conduct.it may well be a superceding act
which breaks up the causal nexus
i. This is a jury issue
e. to carry the burden of proving a prima facie case, the P must generally show that the Ds negligence was a
substantial cause of the events which produced the injury. P need not demonstrate, however, that the
precise manner in which the accident happened, or the extent of injured, was foreseeable.
f. Contactors negligence was factor in causing exactly the type of harm they had duty to protect against

Superceding Cause- is an act of a 3rd person or other force that cuts off a negligent actors liability for his own
antecedent negligence
10. Restatement- Considerations Important in Determining Whether an Intervening Force is a Superceding Cause
i. Its intervening brings about a different harm than would have
occurred
ii. That its intervening appears to be extraordinary w/regards to the
normal view of the circumstances (when you step back)
iii. That the intervening cause is acting independently of s action, and
is not a normal result of s action
iv. Operation of the intervening force due to a third persons act or
omission
v. If the intervening act force is due to the act of a third person which is
wrongful to the other and and as such subjects the third person to
liability
vi. Degree of culpability of a wrongful act of a third person that sets the
intervening force in motion.
11. Natural occurrences (acts of God) cut off proximate cause liability
9.

12. Watson v. Kentucky and Indiana Bridge R.R.


RULE: Willful/wanton intervening cause with could bar recovery.
RR negligently spilled gasoline, defendant might have deliberately thrown match.
a. Do we want to put the risk of arsonists on people? Have to protect against malicious and further
negligent people? (Twerski says no).- here D isnt held liable
b. Risk and plaintiff are foreseeable, only thing not foreseeable is the manner of the harm.
c. Could say this is just a how and then there ought to be liability. But there is a policy question do we
want to hold a negligent person responsible for an arsonist?
d. one is liable for an injury caused by the concurring negligence of himself and another to the same extent
as for one caused entirely by his own negligence
e. mere fact that concurrent cause or intervening act was unforeseen will not relieve the D guilty of the
primary negligence from liability, but if the intervening agency is something so unexpected or
extraordinary as that he could not or ought not to have anticipated it, he will not be liable and certainly he
is not bound to anticipate the criminal acts of others by which damage is inflicted, and hence is not liable
therefore.

26

13. Fuller v. Preis CAN BE LIABLE FOR AN UNFORESEEABLE SUPERSEDING CAUSE- SUICIDE
Car accident seizures suicide.
a. Suicide is an intervening superseding cause. Is it foreseeable?
b. Suicide was an irresistible impulse makes it involuntary.
c. Goes to a jury not a d.v.
d. Twerski says yes. If you push a mentally ill person to suicide you are liable.
e. Combo of thin skull and something else.
f. Varying facts will have a different result He had 3 seizures the day of his suicide, if he had had them a
month before could be a different outcome.
14. Barry v. Quality Steel Products (2003)
a. Doctrine of superceding cause no longer plays a useful role in our common law of proximate cause and
the doctrine of superceding cause should not have been presented to the jury in this case
b. If the intervening cause is within the scope of the risk, there ought to be liability (these are proximate
cause issues)
15. Wagner v. International Railway (1921)
IF YOU CAUSE DANGEROUS SITUATION, YOU CAN BE LIABLE FOR HARM THAT RESULTS FROM
RESCUE.
Train driver didnt close door, cousin fell out, cousin went to rescue, fell off plank and died.
a. The tort is still playing out.
b. Rescuers are in the scope of the risk (are on the risk list).
c. The wrongdoer may not have foreseen the coming of a deliverer, but he is accountable as if he hadwhether or not P acted reasonably during course of rescue is irrelevant
d. Plaintiffs can do something so dangerous to change proximate cause question.
16. Some Rules For Rescue
a. Injuries caused during a rescue must be proximately caused by the original wrong for recovery (jury
question)
b. Rescue doctrine not available to those who come for a reason other than rescue- to see accident or what
not
17. DUTY
17. Hamilton v. Beretta U.S.A. Corp. p. 280
Rule: MARKET SHARE CANNOT WORK.
Market share liability for hand guns.
a. Risks of handguns are foreseeable, but foreseeability does not determine the duty, it determines the
SCOPE of the duty.
b. No duty to control the actions of another- need relationship to establish duty (master to servant, parent to
child)
i. No duty from limited class of Ps to members of community at large
c. Many cause-in-fact problems and not enough proximate cause to even it out- connection b/t Ds and Ps is
too remote
d. Such broad liability should not be imposed w/o a more tangible showing that Ds were a direct link in the
causal chain that resulted in Ps injuries, and that Ds were realistically in a position to prevent the wrongs
e. Every case depends on circumstances. Example, we hold bartenders responsible for serving visibly
intoxicated people.
f. In DES market share because:
i. Manufacturers acted in a parallel manner to produce an identical, generically marketed product.
Guns not identical.
ii. The manifestation of injury were far removed from the time of ingestion of the product.
Possible to establish relative fault of each manufacturer.
iii. Market share was an accurate reflection of each makers tortious conduct. Every gun is not
equally negligent.
iv. Legislature took off statue of limitations.

27

CHAPTER 6
NON-LIABILITY FOR FORESEEABLE CONSEQUENCES (LIMITED DUTY RULES)

5 categories of limited duty rules: 1) rescue; 2) enabling torts in which one party is held liable for negligently
making to possible for another party to cause injury; 3) the obligations of owners or occupiers of land to those who
enter on the land; 4) pure emotional loss; and 5) pure economic loss.
Enabling leave keys in car ignition and may be liable to plaintiff who is injured by thief. Bartender serves more
alcohol to a obviously drunk patron but social guest has no duty.

1.

No pure duty to rescue. Clear cut line dont allow facts (Lubitz v. Wells). No fuzziness and no violins.
a. No duty because:
i. What would the duty be? Reasonable person standard?
ii. How many defendants would you have?
iii. Causation problems.
iv. How to articulate the rule?
b. You start a rescue might make things worse or deter others. We impose a duty.

2.

Yania v. Bigan
NO DUTY TO RESCUE.
Bigan urged Yania to jump in pool, he did, drowned, no duty to rescue.
a. a person is not negligent for taunting someone into doing something destructive or failing to come to their
rescue when they put themselves in a dangerous situation.

3.

Erie R. Co. v. Stewart ONCE YOU START A RESCUE, WE IMPOSE A DUTY.


RR had a watchman, and he was gone.
a. RR assumed a duty to have a watchman. Therefore created a sense of reliance.
b. Change facts (person over road 1st time) could have different outcome.

4.

Lacey v. United States


a. Coast guard started a rescue and did not finish.
b. Court dismissed the complaint because no other potential rescuers were deterred.

5.

L.S. Ayres & Co. v. Hicks


INSTRUMENTALITY UNDER YOUR CONTROL AND YOU ARE NOTIFIED, LIABLE FOR ADD ON
DAMAGES THAT RESULT FROM YOUR FAILURE TO INTERVENE.
Little kid got hand caught in escalator.
a. No negligence in operation and design of escalator.
b. Court says no duty to anticipate accidents does not make sense, still an inherent risk in escalators.
c. Instrumentality under the defendants control and defendant liable for any damages that were a result of a
failure to act within a reasonable amount of time.

6.

J.S. and M.S. v. R.T.H.


STATUTE USED AS BACKDROP TO IMPOSE A DUTY.
Husband sexually abused girls and wife knew and never reported.
a. When statute gets past duty, there might be a duty to rescue in these terrible sexual assault cases.
b. Court did not follow statute didnt say she had to report to Division of Youth and Family services.
c. Will we impose duty on next door neighbor? Twerski has great doubt.

7.

Tarasoff v. Regents of California PSYCHIATRISTS RELATIONSHIP TO A DANGEROUS PATIENT GAVE


RISE TOA DUTY TO WARN THE PATIENTS INTENDED VICTIM THAT THE PATIENT HAD
THREATENED TO KILL HER.
Boy threatened to kill girl, then killed girl, therapist reported to the police, but did not warn parents of girl.
Therapist held liable.

28

a.
b.
c.

Special relationship gave rise to a duty to prevent one person from injuring another.
Not a pure no duty because of doctor-parent relationship and expertise.
Once a therapist does in fact determine (know) or reasonably should have determined (should have
known) that a patient poses a serious danger of violence, bears a duty.
d. Court was ready to impose a duty because someone had died.
e. Psychiatrists now sanitize records. (2 sets of books, one discoverable, one not [wouldnt put the proof of
intent to murder in the discovery books, b/c it would crucify the Dr.])
f. Usually limited to a specifically identified third party.
g. Therapist has a duty to an unknown party, and his obligation to maintain confidentiality conflicts with this
legally-created duty to the third party.
h. Tough Question. This was a rare case so imposed a duty.
i. 3 principles for enabling tort:
i. Defendant strategically placed to take precautions to reduce risk of victim. Social host not
enough.
ii. Existence of a pre-existing relationship (not in Tarasoff).
iii. Cost of precaution is slight just a phone call.
8. Baker v.. Fenneman (Taco Bell)- ONCE YOU START TO AID, NEED TO CONTINUE TO DO SO
Baker entered taco bell, while in there fell backward, hit head, was knocked unconscious and had convulsions. He
stood up, fell again and was knocked unconscious.
a. Does biz have duty to provide reasonable care to patron who was injured through no negligence of the biz
and when injury was not caused by instrument of the biz?
b. Yes, biz had a duty to take reasonable action to give aid to patron after he is injured and to care for him
until he could be cared for by others.
c. a possessor of land who holds it open to the public is under a similar duty to members of the public who
enter in response to his invitation
9.

Stockberger v. US- EMPLOYER DOESNT HAVE DUTY OF CARE TO EMPLOYEE DRIVING HOME
FROM WORK
P has hypoglycemic eps- coworkers would urge him to eat when they saw him like this, on day of death he insisted on
driving home, despite coworkers trying to get him to stay longer to recover- nobody took his keys or offered to drive
him home
a. Indiana has no duty to care, therefore employer not liable
Limitations on recovery for pure economic loss. Should there be a bright-line (WHAM!) rule or a case-by-case rule?
Shortfalls for deterrence, but the defendant is probably threatened with liability for property.
Can have business-interruption insurance.
10. State of Louisiana v. Testbank
RULE: NO RECOVERY FOR PURE ECONOMIC LOSS. LEAVE FOR CONTRACT.
PCP spill and great economic loss for seafood enterprise.
a. Need a WHAM rule because it would be arbitrary any other way.
b. Business losses are infinite, unforeseeable and unpredictable. Need an end to damages. Take out
business-interruption insurance which is in proportion to the amount of business you do.
11. People Express Airlines, Inc. v. Consolidated Rail Corp. Chemical escaped railway tank car, surrounding area evacuated, airline economic loss.
a. Court looks at proximate cause (does not use no duty rule)- foreseeability is important- should have
known test
b. Twerski firm no duty rule except at EXTREMES.
c. Usually, when recovery is permitted, the defendant risked physical harm to the plaintiff or a third party.
Pure Emotional Loss Bystander Emotional Distress Cases
Worst kind of cases no difference between seeing injury and arriving 10 minutes later.

29

Originally general rule of no recovery because floodgates of litigation would be opened.


Impact rule
a. can be no recovery for fright alone w/o impact
b. no recovery for any resulting physical manifestations of the fright- courts reluctant to give this up
- might be different from negligent infliction of emotional distress
- first, you have to figure out what kind of injury you are going to recognize
a. emotional distress
b. serious emotional distress
c. physical manifestation
- its one thing if negligence is toward P, but what if its toward another- what if its just a bystander
a. when do you have duty to bystander?
b. What is the relationship to the victim?
NY Zone-of-danger Rule defendant owes bystanders a duty because they are within the area of the risk
created by his conduct, and hence injury to them is foreseeable.
a. located at scene, not at some distance away
b. whether shock resulted from direct emotional impact upon P from observance of event, as contrasted with
learning about it from others after its occurrence
c. whether P and victim were closely related
d. also was accident and harm reasonably foreseeable
CA Dillon Rule defendants have a duty to avoid infliction of emotional distress that is reasonably foreseeable,
including infliction of such distress on indirect victims. 3 factor test:
a. Located at scene of accident
b. Shock resulted from direct emotional impact and contemporaneous observance
c. Relationship between plaintiff and victim.

12. Daley v. LaCroix IF YOUR NEGLIGENCE CAUSES EMOTIONAL DISTRESS, AND THAT CAUSES PHYSICAL INJURY
THAT IS FORESEEABLE RECOVERY.
Def. drove into electrical wire explosion in plaintiffs house.
a. Physical injury operates as proximate cause (Issue remans: What needs to be foreseeable? The emotional
distress or the physical injury?)
b. Physical injury tells if there is a duty or not.
c. Only some courts.
d. where a definite and objective physical injury is produced as a result of emotional distress proximately
caused by Ds negligent conduct, the P in a properly pleaded and proved action may recover in damages
for such physical consequences to himself even where there was no physical impact
i. Still need to prove proximate cause
ii. And natural result of (to person of ordinary experience and judgment)
13. Thing v. La Chusa CALIFORNIA SAID CONTEMPORANEOUS MEANS YOU WITNESSED THE
ACCIDENT.
Car accident, plaintiff did not see accident but saw son badly injured- need to actually be at scene when accident
occurs to recover for emotional distress
a. Relying on foreseeability is inadequate.
b. Relying on familial relations is arbitrary.
c. Revert to old rule.
d. Loved ones getting hurt is part of the human condition. Cannot hold defendant liable for the general
trauma of life.

30

Harm to Unborn Children: Unborn children are persons capable of being physically harmed for the purposes of bringing an
action to recover for civil damages later on, after the child is born with physical injuries caused prenataly by a wrongdoers
conduct. Parents may also be injured by the same conduct that harmed the fetus.
14. Werling v. Sandy
RECOVERY FOR A STILLBORN CHILD IF AFTER POINT OF VIABILITY.
Wrongful death of a still born child.
a. An unborn fetus can be considered a person if it is past point of viability.
b. Parents bring suit for child.
c. NY no cause of action for child, mother and father have a cause of action for mental distress. Prevents
duplicate recovery.
15. Some states say recovery even when fetus not viable. NY requires live birth. Did not allow recovery for car
accident and then stillbirth of twins. Said mother may sue for injuries she sustained and the father for loss of
consortium so there would still be compensation.
16. - What if the child injured in utero is later born alive suffering a disability caused by the Ds negligence?
a. Every US jurisdiction allows recovery in such a case whether or not the fetus was viable at the time of the
injury in utero
17. What about recovery when child injured in utero and later born alive suffering a disability?
Wrongful birth/Wrongful pregnancy brought by parents alleging that, but for the s negligence, the parents
would have terminated the pregnancy or the mother would have never become pregnant at all. Sue doctors for lost
wages, medical expenses and loss of consortium, physical pain and suffering and cost of raising child.
a. Wrongful life claims brought by child
b. Wrongful birth claims brought by parents- we would have aborted
c. These arent a matter of the doctor causing the problems, but they either failed to inform parents about the
problems or failed to discover the defect
d. How can you prove parents would have chosen abortion- its not what reasonable person would have
done, its what specific person would have done
e. There should be a cause of action for informed consent in these cases
f. NY- parents can only recover cost of raising the child, no emotional distress
g. NY- kids can recover nothing
18. Procanik by Procanik v. Cillo INFANT MAY RECOVER EXTRA MEDICAL EXPENSES BUT NO RECOVERY FOR EMOTIONAL
DISTRESS DUE TO DISABILITY.
Wrongful birth parents claim doctor failed to warn of possible birth defect and caused them to either not get
pregnant or not to terminate the pregnancy.
Wrongful life Claim brought by infant through parent saying, I would have been better off dead.
a. Mother not diagnosed with German measles and child born wth congenital rubella syndrome.
b. Allow recovery for extraordinary medical expenses attributed to affliction.
c. No recovery for emotional distress or imparied childhood.
CHAPTER 7
OWNERS AND OCCUPIERS OF LAND
1.

Categories for court to decide what category, not jury


a. Trespasser
i. Once you know they are there, have to warn.
ii. Cannot maintain conditions that are wantonly or willfully dangerous.
iii. Exception Attractive Nuisance Doctrine foresee trespasses and then reasonable care.
iv. Children- stricter duty: (but must satisfy all the conditions)
1. when know or should have known that kids might trespass
2. when know or should have known there is risk of death or serious harm
3. when kids b/c of youth dont realize risk
4. burden of eliminating the dangers are slight compared to risk to kids (risk-utility
formula)

31

b.

c.

5. possessor fails to exercise reasonable care to eliminate the danger


Licensee/Social Guests
i. Are on duty with consent of owner, but there for their own purposes- guests fall into this
ii. No duty to inspect or fix known dangers
iii. Have to warn of unseen dangers.
Invitee
i. Business Relationship
1. Highest class of entrant
ii. Open to the public
iii. Full duty of reasonable care

2.

Gladon v. Greater Cleveland Regional Transit Authority STATUS OF ENTRANT DETERMINES STANDARD.
Drunk guy on platform (invitee) on tracks (trespasser).
a. Jury instructions said ordinary care. WRONG.
b. Once he is on the tracks he is a trespasser, only have to not be willful or wanton.
c. After you discover you have to act reasonably? Was discovering the shoe discovering him? Was drivers
conduct reasonable?
d. The moment he becomes a trespasser the duty goes way down, and going 5 MPH over the speed limit
really doesnt matter.
i. Sell Tupperware go from licesee to invitee. Is this really fair?

3.

Rowland v. Christian COURT ADOPTS STANDARD OF REASONABLE CARE, NO MORE CATEGORIES.


Landlord knew of crack in faucet (hidden danger), guest hurt himself.
a. Even if licensee there would be a duty to warn of hidden dangers.
b. Reasonable-conduct-under-the-circumstances standard adopted makes things murky for jury. Should
you check the bathroom? Not likely that the court will direct a verdict on this.
c. Now look at factors to determine scope of defendants duty the foreseeability of the harm, the
connection between s conduct and the s harm, and the moral blame attached to the s conduct.

4.

Carter v. Kinney
OLD RULES ARE SIMPLE. SOCIAL GUEST DUTY TO WARN OF HIDDEN DANGERS YOU KNOW
ABOUT. WHAM!
Bible study, broke leg on ice that owner did not know about- Ds recd no tangible benefit from having bible study
at their home
a. Old Rules social invitee only duty to warn of dangers owner knows about.
b. New rules full duty of reasonable care.. and what is that?
c. Court rejects new rule in order to maintain some firm rules. They cut very hard and could produce unfair
results, but work 90% of time and that is good enough.
d. Sometimes we prefer a firm WHAM to violins.

5.

Taylor v. Olsen
Categories of urban and rural similar to trespasser categories.
a. No causation even if breached a duty to inspect he could not see the damage because tree was rotting
in the inside.
b. Do BPL if the burden is too great courts will direct a verdict.
c. Duty to inspect trees in urban areas, no duty to inspect trees in rural areas.
d. Court could still direct a verdict.
e. Generally, a possessors duty of reasonable care will arise from actual knowledge of danger

6.

Sargent v. Ross
Court makes a new standard for landlords must exercise due care under all circumstances.
a. s daughter fell to death from stairway
b. Old Liability for landlords:

32

c.
d.
7.

i. Warn of hidden dangers (not a full duty of reasonable care).


ii. Full duty for common areas, and not for private areas.
iii. Liable if premises are negligently repaired.
This case, the staircase not a common area but the court gives the landlord full duty of reasonable care.
Court is aware of your right to be a slob in your house, but you have to take reasonable measures to
protect injuries to those on your property that can be reasonably foreseen.

Prosecai v. Wal-Mart Stores, Inc.


Duty of landowner to prevent crimes that occur on or near his property Balancing Test.
a. Woman lost 19 grand in jewelry.
b. Causation is impossible in some of these cases would security make a difference?
c. 4 approaches:
i. Specific harm rule no duty to protect unless aware of specific harm.
ii. Prior specific incidence test must establish prior crimes to get in door of court, and then ask
reasonableness question.
iii. Totality of the Circumstances BPL (more likely done by jury)
iv. Balancing Test foreseeability v. burden of duty to protect BPL. (more likely done by judge)
d. Court say balancing test and not negligence test because they are going to sit on the issue and not impose
a duty unless something is extremely bad, and then court will direct verdicts.
CHAPTER 8:
AFFIRMATIVE DEFENSES

Defenses Based on Plaintiffs Conduct

- D may claim P failed to act reasonably

- D may claim that P assumed the risk of injury

- D may claim that P has failed to undertake ameliorative action that would lessen any injury that P may have
incurred

- what if both are but-for causes?


Court developed ways that P can still recover some if even if they are contributorily negligent

- Last Clear Chance


if D had the last opportunity to avoid the peril- P could still recover
based on the idea that D would be more at fault- this isnt always the case
1.

Contributory negligence common law is negligent and his negligence is the proximate cause of the injuries
is totally barred from recovery.
a. Limits on Contributory Negligence
i. Last clear chance had an opportunity to prevent harm and this wipes out s effect of
contributory negligence.
ii. Plaintiff intentional tortfeasor, s negligence is wanton or reckless, and strict liability,
criminal activity (dont know how serious the crime has to be).
Butterfield v. Forrester- P was riding his horse too fast and ran into a pole that D had put up too close to the road
if a person riding with reasonable and ordinary care could have seen and avoided the obstruction, the jury
should find for the D
2 but-fors which bars recovery under contributory negligence

2.

Comparative Negligence Divides liability between and in proportion to their relative degrees of fault. s
recovery is reduced by a proportion equal to the ratio between his negligence and total negligence. Systems:
a. Pure: Damages exactly recoverable. can be 99% at fault and recover 1%.
b. Modified:
i. Over 50% wins unless the s negligence is GREATER THAN that of the .
ii. Under 50% - wins if s negligence is LESS THAN s negligence. In 50-50 cases the will
lose.

33

c.

iii. Majority rule is as long as Ps fault is less than the combined fault of the Ds
Restatement will look at the strength of the causal connection between the persons risk-creating
conduct and the harm.
i. They are comparing proximate cause. Permit jury to take into account where on the risks were
on the risk-list.
ii. The how will not get a off the hook but it will affect the percentages of fault that the jury
assigns.
iii. RESULT: based on comparative fault, you must find both parties negligent and the proximate
cause of the harm
1. This means that if P. D1 and D2 all have some fault, the jury must decide where certain
risks fall on a proximate cause scale (ie parties risk list), and then take it all into
account when comparing fault

3.

McIntyre v. Balentine Tennessee adopts comparative fault judicially. Less than Rule.
a. 2 drunken drivers get into an accident this is the 50-50 rule so no recovery.
b. Now fault of multiple defendants can be divvied up so no more joint and several liability.
c. can recover is his fault is less than the combined negligence of both s.
d. Last clear chance rule is gone.
e. Seems that Tennessee law is wrong because it removes s percentage of fault from each , and that is
double counting. But that is how the law works.
f. Renders the doctrine of joint and several liability obsolete. We can now assign percentages to all of the
parties and no one will pay more than the percentage of their fault as opposed to cause.

4.

In an emergency situation, can protect his property or person, unless the risk involved is out of proportion to the
right he is protecting. If his decision is unreasonable, it will constitute assumption of risk. Ex. Guy runs into
house to save dog no contributory negligence.

5.

Alami v. Volkswagen of America, Inc. (NY 2002)


DWI (crime) Use comparative fault for Add on damage even though it cheats comparative negligence.
a. DWI car crash into a pole and the defect in the vehicle enhanced the injury. VW not responsible for
crash, just add on.
b. Barker/Manning Rule if was involved in a crime than crime will totally bar from recovery- get rid
of this
c. There is an argument that VW is responsible because has a job to protect when accidents happen
regardless of how they happen.
d. Just b/c P was engaged in illegal activity doesnt mean the D wasnt at least partially at fault and
shouldnt be held liable- use comparative fault- if VW did defectively design the Jetta as asserted by Ps
expert, it breached a duty to any driver of a Jetta involved in a crash regardless of the initial cause
e. Court of Appeals allowed recovery but on comparative negligence.
f. Dissent would not allow s suit, now burglars will recover for negligence.
g. Restatement took the position that s negligence should not bar and lost the issue.

6.

Assumption of the Risk. knew of the specific risk (not should have known and the risk assumed must be
the same risk that caused the harm) and freely and voluntary agreed to bear that risk.
a. Express contract in advance. waives the s negligence.

7.

Jorst v. DAmbrosio Bros. Investment Co.


Courts can choose not to recognize a waiver if they want to.
a. Parents signed a waiver for daughter to participate in horseback riding. She hurt herself when foot got
caught in pipe. Court said release did not include premise liability because not reasonably foreseeable..
b. Three requirements for release of liability:
i. Release must be clear, unambiguous and explicit in expressing intent and must be simple so
people can understand.
ii. Act of negligence must be reasonably related to the object or purpose of release.

34

c.

iii. Release must not be contrary to public policy.


It must be reasonably foreseeable to the signer of what is encompassed in the agreement. Objective
standard this release was not about premises, but rather horses- reasonable to expect premises to be safe

8.

Implied Assumption of the Risk Old, assumption of the risk a complete bar.
a. If there is no negligence or no breach of duty than do not talk about affirmative defenses. Must make out
a prima facie case first.
b. Format for deciding AOR cases:
i. P must make out a prima facie cause for liability (i.e. D has to have a duty to the P)
ii. The court will then examine whether the P voluntarily assumed a known risk.
c. General Rules:
i. Landowners have no duty to protect social guests, they need only warn. Social guest has
assumed risk of normal circumstances.
ii. In NY and CA, landowners have a duty to make things reasonably safe 9muyst do Learned Hand
formula including foreseeability)
iii. In situations where landowner is required to protect by statute, then court does a regular
negligence analysis of high burden v. low probability and low gravity for areas not protected by
the statute (i.e. if you are one foot over the line where a screen is required at a baseball field)
d. Fleming James
i. Do not need assumption of the risk as a complete bar. This separate defense serves no purpose.
ii. Was negligent? If yes use comparative fault.
iii. Either you dont need it, or its wrong, or you can use comparative negligence.
e. Twerski he is wrong. Jamess theory does not take into account the risk-taker and even if someone is
negligent, you do not want to protect the risk-taker. You do not want to put the risk of risk-takers on
landowners who might be negligent. Pool hypo.
f. Blackburn v. Dorta Florida merges contributory negligence wit the affirmative defense of assumption of
risk.
i.
g. Now we use DUTY to avoid assigning liability when a risk-taker sues.

9.

Turcott v. Fell
Famous horse jockey sues and court says NO DUTY. Eliminate assumption of the risk and you have to jimmy the
case to get a result.
a. Traditionally, used assumption of risk but no longer an absolute defense with comparative fault.
b. Clear that ct does not want to recover. So say NO DUTY. Only have a duty of care not to be willful or
wanton. experienced and could see problem of cupping.
c. P alleged that track was negligently maintained- but court held that in professional sporting contests, if a
participant makes an informed estimate of the risks involved in the activity and willingly undertakes
them, then there can be no liability if he is injured as a result of those risks.
i. Participants have consented to risks by participation in activity
ii. There is no duty of reasonable care, there is only a duty to not be grossly negligent

10. Doctrine of Avoidable Consequences Comes into play AFTER the legal wrong has happened (comparative
negligence happens before). may fail to take reasonable steps to avoid aggravation of injury. Policy question:
Should we take all recovery away from ? Or should we do comparative fault?
a. Failure to wear a seat belt NY, NJ, and CA you failed to mitigate your damages. But this is a failure
BEFORE the accident and almost all avoidable consequences are for AFTER the accident. Saying, Now
behave! Take your meds.
11. Bryant v. Calantone (NJ 1996)
a.
b.
c.

He is not negligent for not taking medicine before the dental procedure, but AFTER he did not ask for
medicine or call his doctor.
Court remands and asks if this failure to act after the procedure caused an ADD ON? Hard to determine.
Restatement and Twerski would recover 100% of damages that would have resulted even if had
taken due care. Do comparative fault for the add-on.

35

d.

What if all damage is add on reducing recovery completely seems grotesque.

HYPO: P in car accident, D at fault, P gets $10,000 worth of injuries. For not taking meds after the accident, P
becomes seriously ill and injuries are now $500,000. D is liable only for initial $10,000 b/c the rest was
avoidable by the P. Court generally doesnt want to dump all avoidable consequences on P so usually says
comparative fault and lets the jury decide/apportion percentages.
HYPO: P was hit by a drunk driver, not wearing a seatbelt, and is thrown from car. Expert testimony says had
P been wearing a seatbelt would only have been $100k in damages, b/c not wearing it was $1,000,000
o under the pure avoidable consequences doctrine, P only recovers $100,000 b/c the $900k could have
been avoided. TWERSKI thinks this is unfair
o under new Restatement and Bryant analogy, P gets regular $100,000 with no dimunition. Regarding
the remaining $900,000, you apply comparative fault/ hold comparatively negligent
12. Non-Conduct Based Offenses
a. Immunities
i. Interspousal- courts have not been willing to treat the pain accompanying marital infidelity and
other forms of emotional distress within the marriage as sufficiently outrageous to support a
mental distress claim
ii. Parental- court recognizes 2 situations in which parental immunity from tort actions should
remain intact:
1. Where the negligence involves parental discipline over the child
2. Where the negligence involved exercise of normal parental discretion over such matters
as providing food, housing, and medical services
iii. Charitable Immunity- not as much now
iv. Govt Immunity
1. Discretionary immunity (federal)
Tippett v. US- if actions are discretionary, not held liable
Riss v. City of NY- P was being threatened by man, told the police, but they didnt do anything to protect her and
she was disfigured after attack
- there is no warrant in judicial tradition or in the proper allocation of the powers of govt for the courts, in the
absence of legislation to carve out an area of tort liability for police protection to members of the public.
CHAPTER 9
JOINT TORTFEASORS
In 3 situations, a tortfeasor can be held responsible to pay all of the Ps damages even though other tortfeasors are also
responsible for the harm:
1. Concerted Action clearly acted negligent together ex drag-racing.
a. Herman v. Wesgate 2 defendants threw Herman off a boat against his will and he was injured.
b. Courts have said you need more than just parallel activity to constitute conspiracy (if you can prove
conspiracy you are a joint tortfeasor). Ex. Cigarette litigation they all worked to hide info but this was
not sufficient.
c. Restatement - 876 persons acting in concert when:
i. Act in concert. There is an agreement that does not need to be expressed in words, may be
implied.
1. Drag racing.
ii. Knows the other conduct constitutes a breach of duty and gives SUBSTANTIAL assistance or
encouragement to the other.
1. Tell someone to deliver a message to commit a tort. Messenger is not liable.
iii. Gives substantial assistance to the other in accomplishing a tortuous result and his own conduct,
separately considered, constitutes a breach of duty to the third person.
2. Liability by action of law employer liable for actions of employee they are joint tortfeasors. Landowner
may be held liable for independent contractors, car owner may be held liable when loan out car
3.

Concurrent Tortfeasors cause a single indivisible injury.

36

a.
b.
c.
d.
e.

f.

50 states had joint and several liability comparative fault 30-32 states have done away with joint
and several liability.
TWERSKI likes joint and several b/c under comparative fault- P can get screwed- what if main guy at
fault is broke? Under joint and several- thats fine, just go after the other guy
Breakdown:
i. 15 states traditional joint and several liability Twerski says will switch.
ii. 16 states abolished it. (Big money s are happy).
Different approaches:
i. Apply j&sl for ECONOMIC harm but not non-economic (pain and suffering).
ii. Threshold j&sl if the is more than x% of fault (NY is > 50%).
NY:
i. First meet threshold of over 50% at fault. If you are under 50% you only pay your damages for
non-economic harm.
ii. Then you are liable 100% only for economic harm.
iii. However, has j&sl for automobile accidents.
Different schemes hurt s enormously and affect willingness to settle:
i. Common law settlement with 1 joint tortfeasor released all others from liability greatly
discouraged settlements.
ii. Restatement release of one tortfeasor does not release the others unless the release
specifically so provides.
iii. If you permit contribution you kill settlement. v. D1 (recovery) D1 v. D2 AND v. D2
effect of collusive settlements.
iv. NY When settles with D1, he settles EITHER the dollar amount of the settlement or the
percentage of fault he is found later whichever is greater..
1. deters collusive settlements.
2. But does not want to settle wants to see how much a jury will give him.

4.

American Motorcycle Assn. v. Superior Court of Los Angeles County


Comparative negligence does not abolish joint and several liability of concurrent tortfeasors each but for and
proximate cause remains individually liable 100%.
a. The suffered an injury 100% caused by these defendants so should be able to recover 100% of his
damages.
b. AMA negligently administered race and wanted contribution from parents.
c. A concurrent tortfeasor whose negligence is a proximate cause of an indivisible injury remains liable for
the total amount of damages, diminished only in proportion to the amount of negligence attributable to
the person recovering.
d. P isnt required to prove that a tortfeasors conduct was the sole proximate cause of the injury, but only
that such negligence was a proximate cause.

5.

Michie v. Great Lakes Steel Division, National Steel Corp (1974)


If there is no possibility of dividing the injuries than make them jointly and severally liable.
a. Multiple corps discharge pollutants and separation of pollutants and causation of injuries is impossible to
analyze (indivisible injury).
HYPO: P injured in a car crash, and is the passenger of a GM car. Door lock in GM(D1) car was defective and
P is thrown from car. D2 is speeding and causes the accident. $5 million injury. Expert testimony says that had
P not been thrown from car, would have been $1 million. Therefore, GM is responsible for $4 million.
However, injury is not entirely GMs fault. Both D2 and GM caused them b/c w/o the original accident there
would have been no add-on. Regarding the $ million, GM and D2 are joint tortfeasors. So then you look to the
statute. If joint and several state, then they can get GM for the entire $4million. If several liability state, then
jury must apportion fault regarding the add-on. The problem is that the jury could come back with almost
anything (including barely assigning much fault to GM who technically caused the add-on and is most capable
of paying).

6.

Dillon v. Twin State Gas & Electric Company (NH 1932)

37

a.
b.
c.
d.
e.

Indivisible injury fell on wires and then fell into water do not know what caused the death.
Do we impose liability for a 4-5 second life expectancy?
Court remands question of fact if he would have fallen into the water and just received serious injuries
than damages can be awarded.
When the danger is great and the wrongful conduct of the injured person is not serious, it is reasonable
for the law to find a relationship and to impose a duty of protection
A D in his own interest causing dangerous forces to operate or dangerous conditions to exist should
reasonably protect those likely to be exposes to them and not reasonably at fault for the exposure
CHAPTER 10
STRICT LIABILITY

Function of tort law is optimal deterrence deter excessively risky activity so only those losses worth avoiding are
avoided.
Can make a cause of action without proving negligence, but you still have to prove causation and proximate cause.
It just does not matter if the actor exercised reasonable care.
Should only be imposed where a negligence scheme wont be sufficient to produce the right amount of activity
because we dont want to over deter.
Those who engage in certain activities do so at their own risk and have to pay for any damages that have
foreseeable results even if such activity has been carried out in the most careful way.
Res ipsa is rebuttable, this is not.
Assumption of the risk is a defense.

ANIMALS:
3 categories
1. livestock- strict liability
2. wild animals confined- strict liability
3. domesticated animals- generally not strict liability, unless you know of your animals
propensity for violence
1. Bard v. Jahnke- P hired to repair cow mattress, cows wandered freely where he was working, a bull attacked himbull had no prior vicious propensity
a. Cant collect b/c D had no prior evidence of Freds vicious propensities.
ABNORMALLY DANGEROUS ACTIVITIES:
2. Fletcher v. Rylands Property owner had unnatural reservoir which leaked and held liable.
a. The person who for his own purposes brings on his lands and collects and keeps there anything likely to
do mischief if it escapes, must keep it in at his peril, if he does not do so, is prima facie answerable for all
the damages which is the natural consequence of its escape.
3.

Turner v. Big Lake Texas, need to have reservoirs, no strict liability because there was not negligence.
a. where usage is natural or necessary and common use of land- negligence liability applies

4.

Indiana Harbor Belt RR v. American Cyanamid Co. POSNER said negligence system is adequate and will not impose liability until someone shows him that
negligence is inadequate.
a. Cyanamid loaded chemical into tanker transported to RR leak pollution and RR wants recovery
for clean up.
b. Not liable because only put chemical into tanker and that is not highly dangerous, and he is not convinced
that the laws of negligence are inadequate to control the shipping of chemicals.
c. Leak in this case was caused by carelessness, not by inherent properties of chemical- so negligence works
just fine
d. Restatement 520
i. Risk of harm great.
ii. Harm that would ensue if the risk materialized could be great.
iii. Yet accidents could not be prevented by the exercise of due care.

38

e.
f.
g.
5.

iv. Activity not a manner of common usage


v. Activity inappropriate to place it took place.
vi. Risk of serious harm to others could have been dispersed.
Balloonist fills these criteria and was not held liable.
Strict liability is a very NARROW category, reserved for blasting, crop dusting because there are
alternate means.
Courts dont want to tell people NOT do an activity, just look at factors and tell that he will be held
liable if they decide to do it.

Foster v. Preston Mill Co. Blasting caused mink to kill children. This is outside the scope of the risk so no
strict liability. Strict liability only for the KIND OF RISK that made the activity abnormally dangerous, and not
for abnormally sensitive consequence.
a. The policy of the law doesnt impose the rule of strict liability to protect against harms incidental to the
Ps extraordinary and unusual use of land.
b. Proximate cause issue- no proximate cause here- too far removed
Chapter 11:
PRODUCTS LIABILITY

1.
2.

3.

ATTACK:
a. Determine type of defect.
b. Analyze in terms of negligence and strict liability.
HISTORY:
a. MacPherson v. Buick You no longer need privity.
b. Uniform Commercial Code contract action Implied Warranty of Merchantability warned that the
product is reasonably fit for the ordinary purpose of which its goods are used. Came with disclaimers and
statute of limitations.
c. Create TORT of selling a defective product statute of limitations started to run when defect discovered.
Statute said no privity.
d. Actually went back to fundamental negligence.
TWERSKI REORGANIZES PRODUCTS LIABILITY
a. Manufacturing defect compare defective product with original blueprint and sue! Need to show that
product departed from intended design and this departure caused s injury. Strict liability circumvents
the problems of proof because there is no need for the plaintiff to show that the manufacturer used
improper materials or failed to inspect.- doesnt have to be negligent to be liable
b. Design Defect negligence - design with foreseeable risk or harm could have been reduced by adoption
of reasonable alternative. (This is not a departure from the manufacturers design instead the defect is
in the blueprint itself Question is Was this adequately designed?)
i. This is negligence BPL is there a better way to make it without too much of a burden? Do
the risks posed by the design outweigh the benefits?
ii. Usually plaintiff must prove that risk was foreseeable.
iii. Need RAD making a product better could make it defective or raise BPL too much.
1. Should you have made the product better?
2. What should you have done? Need RAD- Restatement basically demands RAD
iv. Factors Considered in determining whether an alternative design is reasonable and whether its
omission renders a product not reasonably safe, factors include:
1. Magnitude and probability of the foreseeable risks of harm
2. Instructions and warnings accompanying the product, including expectations arising
from product portrayal and marketing
3. Relative advantages and disadvantages of the product as designed and as it alternatively
could have been designed may also be considered
4. When evaluating the reasonableness of design alternative, the overall safety of the
product must be considered. It is not sufficient that the alternative design would have
reduced or prevented the harm suffered by the P if it would also have introduced into
the product other dangers of equal or greater magnitude
v. TWERSKI hates consumer-expectations test

39

4.

1. Allows liability without risk-utility stuff


vi. BARKERS TEST: a product is still defective if its design embodies excessive preventable
danger- that is, unless the benefits of its design embodies excessive preventable danger- that is,
unless the benefits of the design outweigh the risk of danger inherent in such design
1. In such cases, the jury must consider the manus evidence of competing design
considerations
vii. Consumers Expectations1. What are the risks?
2. Would a reasonable consumer consider the product unreasonably dangerous?
c. Inadequate Instructions/Failure to Warn Produce seller is relieved of liability for harm caused by the
risk about which an adequate warning or instruction was provided. With adequate info the consumer can
make the choice to proceed with the activity. Warning for everything is impossible it is costly and
people wont read the warnings. Diminishing returns for providing additional information. Warnings
required for non-obvious risk of personal injury.
i. Risk reduction warning.
ii. Consumer choice warning.
iii. Strict Liability courts adopted the presumption of the heeded warning. Burden on to prove
that more likely than not wouldnt have heeded the warning.
Defense
a. Comparative negligence
b. Unforeseeable harm
c. Product misuse cannot be expected to guard against everything but what about foreseeable misuses?
d. Assumption of risk can be a complete defense.
e. Failure to follow instructions.
f. No way for to know of the risk.

5.

Welge v. Planters Lifesavers Co. Res Ipsa- MANUFACTURER DEFECT


More probably than not accidents do not happen unless there is some defect.
a. Bottles dont just break.
b. Woman testified that she carefully used knife to remove label and she did not damage anything.
c. The P, in a products liability suit, is not required to exclude every possibility, however fantastic or remote,
that the defect which led to the accident was caused by someone other than one of the Ds.
d. Doctrine of res ipsa loquitor teaches that an accident that is unlikely to occur unless the D was negligent
is itself circumstantial evidence that the D was negligent.

6.

Prentis v. Yale Manu- DESIGN DEFECT


a. forklift when running out of battery would get surge of power and become dangerous- P was thrown from
P
b. adopt a pure negligence, risk-utility test in products liability actions against manus of products, where
liability is predicated upon defective design
c. in essence, ask jury to consider whether the manu took reasonable care in light of any reasonably
foreseeable use of the product which might cause harm or injury- was this an unreasonably dangerous
product

7.

Vautour v. Body Masters Sports- DESIGN DEFECT ACTION DOESNT DEMAND FIRM RAD PROPOSAL
a. P injured while using leg press- claims should be designed with adjustable stops
b. Court held dont have to have firm proposal for RAD- just knowledge that there is better way
c. To prevail on a defective design products liability claim, a P must prove the following:
i. Design of the product created a defective condition unreasonably dangerous to the user
1. Must be dangerous to an extent beyond that which would be contemplated by the
ordinary consumer who purchases it, with the ordinary knowledge common to the
community as to its characteristics
2. Determine risk-utility balancing test- defective if magnitude of danger outweighs the
utility of the product
ii. The condition existed when the product was sold by a seller in the biz of selling such products
iii. Use of the product was reasonably foreseeable by the manufacturer

40

iv. Condition caused injury to the user


8.

Potter v. Chicago- TWERSKI THINKS THIS IS RIDIC


a. Replaced RAD with Modified Consumer Expectations test.
b. A product is defective if it fails to meet reasonable consumer expectations.
c. TWERSKI idea out of hell - Where do you get the expectations? A company cannot make a product to
meet everybodys expectations.
d. This is strict liability. The issue is no longer- did you make the safest design? The issue is- are consumer
expectations satisfied?
e. Dont need RAD to prevail under this idea
f. Does not care about risk-utility. Does not have causation. And no technological expertise is necessary.

9.

Soule v. GM- DESIGN DEFECT


a. P was in car accident and suffered permanent injury to ankle when floorboard smashed into her feetclaimed injury was not a natural consequence of the accident, but occurred when the collapse of the
Camaros wheel caused the toe pan to crush against her feet
b. Alleged both manu and design defect
c. Issue is whether a consumers expectations or risk-utility balancing test should be used in determining
product liability for design defect
d. Consumers expectations test can only be used in cases in which the everyday experience of the products
users permits a conclusion that the design violated minimum safety assumptions
e. In other cases, esp where evidence is more complicated, risk-utility balancing must be used

10. OBrien v. Muskin Corp- DESIGN DEFECT


a. P dove into pool and sustained personal injuries- claims pool was defectively designed and bore an
inadequate warning
b. Risk-utility analysis is appropriate when the product may function satisfactorily under one set of
circumstances, yet b/c of its design present undue risk of injury to the user in another situation
c. Dont need to have RAD- involved in consideration, but not an absolute determination
d. Need to consider need for that product- dangerous products that are a luxury should be viewed differently
than essential products
e. Even when no alternative method of design, jury can find that risk of something outweighs its utilities
11. Anderson v. Owens-Corning Fiberglas Corp. (1991 CA)- FAILURE TO WARN
a. says you should warn me about dangers attached to using this. This means the has to warn for
everything.
b. Rules of strict liability require a P to prove only that the D didnt adequately warn of a particular risk that
was known or knowable in light of the generally recognized and prevailing best scientific and medical
knowledge available at the time of manu and distribution
i. Thus, in strict liability, as opposed to negligence, the reasonableness of the Ds failure to warn in
immaterial
ii. State-of-the-art evidence can be presented to show lack of knowledge of the dangers.
iii. Are there strict liability or negligence standards?
c. You could warn for everything and nobody will read them.
d. In order for warnings to be effective they have to be selected. Need to make the best possible design and
only warn for the residual.
e. Would the warnings have done any good? Would have acted upon them? Of course says, of course I
would have acted on warning. How can I act on it if you dont give it to me.
f. Most dangerous tool for s.
g. Notwithstanding the defective condition of the product in the absence of adequate warnings, if a
particular user or consumer would have decided to use or consume even if warned, the lack of warnings is
not a legal cause of that Ps harm
What about comparative fault in products liability cases?
design defect and failure to warn are basically negligence anyway
most courts say you should apply comparative fault to products liability cases

41

we shouldnt let Ps get full recovery when they are acting stupid

Chapter 12:
TRESPASS TO LAND AND NUISANCE
A. TRESPASS
a. Liable for trespass even when no harm
b. Liable when acting even on mistaken belief as to ownership (mistake is no defense in trespass actions)
c. If no intent, liability will only attach on the basis of another recognized action
i. Ex. Negligence- car crashes into someones land- was driver negligent?
ii. Ex. Strict liability- if A is using dynamite on his land and rocks are hurtled onto Bs property
B. COMPETING USES OF LAND
a. Whose rights predominate
b. Both parties are affecting the others use of the property
c. Unfortunate that some cases starting calling these particle cases trespass cases (Twerskis opinion)
d. Trespass protects your right to ownership of property
e. Nuisance protects your right right to use and enjoyment of property
f. Particles dont threaten ownership, threaten use and enjoyment
g. How do you balance the rights between the two?
C. NUISANCE
a. Protect ones right to the use and enjoyment of property
b. Unlike trespass, which protects against physical invasion of property by another, the tort of nuisance
protects against intangible invasions of ones land
i. Noise, smoke, etc.
c. Does not have black and white quality of trespass
D. Bradley v. American Smelting & Refining Co.
a. D was aware that the wind, on occasion, caused these emissions to be blown over Ps land
b. P sued for damages in trespass and nuisance from the deposit on their property of microscopic, airborne
particles from Ds plant
c. Intent doesnt have to be required, just has to have been certain that consequences were to result from
their action
d. D had requisite intent to commit intentional trespass, so D can be liable if P suffered actual and
substantial damage as a result
e. If the intrusion interferes with the right to exclusive possession of property, the law of trespass appliesdoesnt have to be a tangible intruding agent
f. In order to recover in trespass for intangible agent, P must show ALL 4:
i. An invasion affecting an interest in the exclusive possession of his property
ii. An intentional doing of the act which results in the invasion
iii. Reasonable foreseeability that the act done could result in an invasion of Ps possessory interest
iv. Substantial damage to the res
g. Recognizes 2 kinds of trespass- not all courts agree with this- most dont
i. Direct trespasses that are subject to the traditional rule that any interference with exclusive
possession constitutes a trespass
ii. Indirect trespasses arising from some form of pollution which require proof of actual harm
E. PRIVATE NUISANCE
a. Must establish
i. Basis for liability
1. Intentional tort
2. Negligence
3. Strict liability
ii. significant harm
iii. unreasonable invasion of Ps land
1. looks at severity of harm to P
2. is this an expected harm from living in industrial society

42

F.

G.

H.
I.

J.

K.

L.

Hughes v. Emerald Mines Corp.


a. Complaint of landowners that a coal company operating on an adjacent property caused the failure of one
water well and the population of a 2nd well located on the Ps land
b. action tried on basis of non-trespassory invasion of anothers land
c. D liable b/c actual physical damage did occur and D didnt fulfill its burden to prove that its acts werent
available or only avoidable at prohibitory expense
Carpenter v. The Double R Cattle Co.
a. Ps live near feedlot and say the smell, pollution etc associated with the growing size of the feedlot is a
nuisance
b. in determining whether a nuisance exists, considerations should be given to such factors as community
interest, utility of conduct, business standards and practices, gravity of harm caused, and the
circumstances surrounding the parties movement of their locations
c. dissent says those directly impacted by the serious nuisance deserve some compensation for the invasion
they suffer as a result of the continuation of the nuisance
Who got there first?
a. Courts sometimes consider who was there first in nuisance litigation
b. Did Ds activity exist when P bought land
AIRPORTS
a. As long as an airport conforms to the fed standards, it cannot be enjoined from continuing its operations
b. When a govt entity operates ann airport such that flight over a Ps land significantly diminishes the value
of the property, the govt must pay for the dimunition in value
i. Cases allowing recovery for airport noise have been few and far between- for the most part cant
show dimunition in property value
ii. Many homeowners moved to the nuisance, in that they purchases homes with full knowledge of
airport noise
c. also use risk-utility balancing
PUBLIC NUISANCES
a. To make out a public nuisance, a P must establish that the Ds conduct involves a significant interference
with public health, public safety, or public convenience
b. For an individual to recover damages for a public nuisance, the P must have suffered harm or a kind
different from that suffered by the general public
NUISANCE THROUGH ECONOMICS: COARSE THEOREM
a. In cases of competing land uses- causation is reciprocal
b. Not a question of who is at fault, but rather whose use should be preferred that presents the problem
c. Under ideal negotiation conditions, the parties will compromise- but sometimes transaction costs are huge
d. When transaction costs prevent parties from negotiating successfully, the law should decide the dispute in
such a way that the outcome is that which the parties would have bargained for in the absence of
transaction costs
REMEDIES
a. Boomer v. Atlantic Cement
i. Cant use personal litigation to direct public objectives greatly beyond the rights and interested
before the court
ii. Court had choice between injunctions and damages- chose damages
iii. Didnt want injunction- damage to Ps property small in comparison with value of Ds operation
Chapter 13:
DAMAGES

A. PUNITIVE DAMAGES
a. 3 reasons
i. deterrence- most tortious acts go undetected, use this to approximate a hypothetical
compensatory award for all the damages the tortfeasors actions have caused in all the cases they
have gone undetected
ii. retribution
iii. compensation- make up to Ps what lawyers take
B. Owens-Illinois v. Zenobia

43

a.

In order to award punitive damages, P must prove the Ds malicious conduct by clear and convincing
evidence before punitive damages can be considered
b. Have to prove wanton and or reckless disregard
C. BMW v. Gore
a. In assessing punitive damages, should consider:
i. Conduct and reprehensibility
1. Economic v. physical damages
2. Ratio b/t compensatory damages and punitive damages (should usually be no more than
10:1 ratio)
3. Should also look at what other possible punishments there might have been for this
behavior (ex. Criminal penalties)
b. in this case, court found that punitive damages awarded were too large
D. State Farm v. Campbell
a. Few awards exceeding a single-digit ratio between punitive and compensatory damages, to a significant
degree, will satisfy due process
b. The wealth of a D cannot justify an otherwise unconstitutional punitive damages award.
c. Can only use punitive damages to punish D for damages done against this P
d. Punitive damages are not a substitute from the criminal process, and the remote possibility of a criminal
sanction doesnt automatically sustain a punitive damages award

44

You might also like